AH1 Eone study

¡Supera tus tareas y exámenes ahora con Quizwiz!

The primary care provider is screening a patient using the CAGE criteria. What will the provider include in this assessment? Select all that apply. a. Number of times per week eaten in restaurants b. Sodium and sugar intake c. Sources of daily dairy intake d. Total number of servings of fruits and vegetables e. Types of meats and proteins

a. Number of times per week eaten in restaurants c. Sources of daily dairy intake e. Types of meats and proteins The CAGE questionnaire is designed to evaluate the intake of saturated fat and cholesterol, so the provider will ask about sources of dairy, the number of times eating foods not made at home, and the types of meats and proteins eaten. --The questionnaire does not evaluate for sodium, sugar, fruits, or vegetables.

Which tests are indicated as part of the initial evaluation for women of childbearing age who report syncope? Select all that apply. a. 12-lead ECG b. Cardiac enzyme levels c. CBC d. Electroencephalogram e. Serum glucose testing

A 12-lead ECG C CBC E Serum glucose testing Initial evaluation for all patients reporting syncope should include a standard 12-lead ECG. Women of childbearing age should have a CBC, serum pregnancy test, and serum glucose testing. --Cardiac enzyme levels are obtained if the patient has cardiac risk factors. --EEG is performed only if there is a concern for seizure disorder.

Which clinical findings are worrisome in a patient experiencing acute bronchospasm, requiring immediate treatment? (Select all that apply.) a. A silent chest after previously wheezing b. Decreasing blood pressure c. Presence of an urticarial rash d. Pulsus paradoxus of 10 mm Hg e. Wheezing on both inspiration and expiration

A A silent chest after previously wheezing =indicates severe spasm and is an ominous sign B Decreasing blood pressure C Presence of an urticarial rash =anaphylaxis (requiring oxygen, diphenhydramine or epinephrine) --A pulsus paradoxus greater than 25 mm Hg is worrisome. --Wheezing on inspiration and expiration is a common finding and not necessarily an emergency.

2. Which are risk factors for the development of cataracts? Select all that apply. a. Advancing age b. Cholesterol c. Conjunctivitis d. Smoking e. Ultraviolet light

A Advancing age D Smoking E Ultraviolet light Most older adults will develop cataracts. Smoking and UV light exposure hasten the development of cataracts. --not Cholesterol and conjunctivitis. (HTN and history of inflammation are, however)

A child is diagnosed as having a congenital cholesteatoma. What is included in the management of this condition? Select all that apply. a. Antibacterial treatment b. Insertion of pressure equalizing tubes c. Irrigation of the ear canal d. Removal of debris from the ear canal e. Surgery to remove the lesion

A Antibacterial treatment D Removal of debris from the ear canal E Surgery to remove the lesion Cholesteatoma is treated with antibiotics, removal of debris from the ear canal, and possibly surgery. --PETs and irrigation of the ear canal are not part of treatment

An older patient develops orthostatic hypotension secondary to antihypertensive medication and asks what measures can be taken to minimize this condition. What will the provider recommend? Select all that apply. a. Crossing the legs when standing up b. Custom-fitted elastic stockings c. Discontinuation of the medication d. Increased physical activity e. Performing the Valsalva maneuver

A Crossing the legs when standing up B Custom-fitted elastic stockings D Increased physical activity --Discontinuation of the medication may make the condition worse. --Performing the Valsalva maneuver will increase intrathoracic pressure and should be avoided.

1. Which factors determine which diagnostic tests should be performed in a presurgical clearance evaluation? (Select all that apply.) a. Patient's age b. Patient's comorbidities c. Previous surgeries d. Surgeon's preference e. Type of anesthetic agent planned

A Patient's age B Patient's comorbidities D Surgeon's preference E Type of anesthetic agent planned The patient's age and comorbidities, surgeon preference, and the type of anesthetic planned all determine which presurgical diagnostic tests will be performed. --The patient's previous surgeries do not determine presurgical testing.

According to current research, which is associated with a decreased incidence of stroke? a. ≧7 servings of fruits and vegetables per day b. B-complex vitamin supplements c. Intensive insulin therapy in type 1 diabetes d. Low-sugar soda e. Mediterranean diet

A ≧7 servings of fruits and vegetables per day C Intensive insulin therapy in type 1 diabetes E Mediterranean diet Individuals without HTN in Sweden who consumed ≧7 servings of fruits and vegetables per day , Intensive insulin therapy in patients with DM1 , and Consuming a Mediterranean diet are all associated with reduced stroke risk. --B-complex vitamins and low-sugar soda have not shown a decreased risk.

Which patient may be given symptomatic treatment with 24 hours follow-up assessment without initial antibiotic therapy? a. A 36 mo old with fever of 38.5° C, mild otalgia, & red, non-bulging TM b. A 4 yo, afebrile child with bilateral otorrhea c. A 5 yo with fever of 38.0° C, severe otalgia, & red, bulging TM d. A 6 mo with fever of 39.2° C, poor sleep & appetite & bulging TM

ANS: A A 36 mo old with fever of 38.5° C, mild otalgia, & red, non-bulging TM Children > 24 months with fever <39° C & non-severe symptoms may be watched for 24 hours with symptomatic treatment. --Children with otorrhea, those with severe AOM, & any children with fever >39° C should be given abx.

3. A non-smoking adult with a hx of CVD reports having a chronic cough without fever or upper airway symptoms. A chest radiograph is normal. What will the provider consider initially as the cause of this patient's cough? a. ACE inhibitor medication use b. COPD c. GERD d. Psychogenic cough

ANS: A ACE inhibitor medication use About 10% of patients taking ACE inhibitors will develop a chronic cough. --COPD, GERD, and psychogenic causes are possible, but given this patient's cardiovascular history, the possibility of ACE inhibitor-induced cough should be investigated initially.

A patient has chronically dry eyes, sometimes with a foreign body sensation, burning, and itching. A Schirmer test is abnormal. What is the suspected cause of this patient's symptoms based on this test finding? a. Aqueous-deficiency b. Corneal abrasion c. Evaporative disorder d. Poor eyelid closure

ANS: A Aqueous-deficiency An abnormal Schirmer test= aqueous-deficient/dry eye. --A corneal abrasion usually causes excessive tearing. --An evaporative disorder is determined by an evaluation of tear breakup time. --Poor eyelid closure causes increased corneal exposure and increased evaporation of tears.

A patient complains of otalgia and difficulty hearing from one ear. The provider performs an otoscopic exam and notes a dark brown mass in the lower portion of the external canal blocking the patient's tympanic membrane. What is the initial action? a. Ask the patient about previous problems with that ear b. Irrigate the canal with normal saline c. Prescribe a ceruminolytic agent for that ear d. Use a curette to attempt to dislodge the mass

ANS: A Ask the patient about previous problems with that ear Before attempting to remove impacted cerumen, the provider must determine whether the TM is intact and should ask about pressure-equalizing ear tubes, a history of ruptured TM, and previous ear surgeries. --Once the TM is determined to be intact, the other methods may be attempted, although the curette should only be used if the mass is in the lateral third of the ear canal.

A health care provider in a clinic finds a patient in a room, unresponsive and pale. Which sign should be used to identify the need to initiate cardiopulmonary resuscitation (CPR)? a. Assessment of gasping breaths or not breathing b. Determination of pulselessness or bradycardia c. Evaluation of peripheral perfusion and level of consciousness d. Obtaining a history of previous myocardial infarction

ANS: A Assessment of gasping breaths or not breathing The AHA recommends initiating CPR if the victim is not breathing or has gasping breaths. --Determination of a pulse in an arrest situation can be problematic and the search for a pulse should not delay chest compressions if the patient is gasping for breath or not breathing. --Evaluation of peripheral perfusion and LOC is not part of the initial assessment and not used to indicate the need for CPR. --A medical history may be obtained after resuscitation is in progress.

A patient reports several episodes of acute vertigo, some lasting up to an hour, associated with nausea and vomiting. What is part of the initial diagnostic workup for this patient? a. Audiogram and MRI b. Auditory brainstem testing c. Electrocochleography d. Vestibular testing

ANS: A Audiogram and MRI An audiogram and MRI are part of basic testing for Meniere's disease. The other testing may be performed by an otolaryngologist after referral.

An adult patient who had pertussis immunizations as a child is exposed to pertussis and develops a runny nose, low-grade fever, and upper respiratory illness symptoms without a paroxysmal cough. What is recommended for this patient? a. Azithromycin daily for 5 days b. Isolation if paroxysmal cough develops c. Pertussis vaccine booster d. Symptomatic care only

ANS: A Azithromycin daily for 5 days Adults previously immunized against pertussis may still get the disease without the classic whooping cough sign seen in children and are contagious from the beginning of the catarrhal stage of runny nose and common cold symptoms. --Azithromycin or other macrolide antibiotics are useful for reducing symptoms and for decreasing the shedding of bacteria to limit the spread of the disease. --Patients should be isolated for 5 days from the start of treatment. --Pertussis vaccine booster will not alter the course of the disease once exposed. --Symptomatic care only will not reduce symptoms or decrease disease spread.

A patient suffers a penetrating injury to one eye caused by scissors. The provider notes a single laceration away from the iris that involves the anterior but not the posterior segment. What is the prognosis for this injury? a. Because the posterior segment is not involved, the prognosis is good b. Blindness is likely with this type of eye injury c. Massive hemorrhage and loss of intraocular contents are likely d. Retinal detachment is almost certain to occur

ANS: A Because the posterior segment is not involved, the prognosis is good Mechanical energy imparted from sharp objects generally results in lacerations, with disruption that is more localized. The prognosis is better if the posterior segment is not involved. --The other complications are more common with globe ruptures.

A patient complains of SOB when in a recumbent position and reports coughing and pain associated with inspiration. The provider notes distended neck veins during the exam. What is the likely cause of these findings? a. CHF b. Hepatic disease c. Pulmonary embolus d. Pulmonary infection

ANS: A CHF CHF causes the symptoms described above, with distended neck veins being a significant finding. --Hepatic disease would also cause abdominal distention with ascites and hepatomegaly. --Pulmonary embolus has marked shortness of breath. --Pulmonary infection causes inflammation and a friction rub.

A provider is recommending a cerumenolytic for a patient who has chronic cerumen buildup. The provider notes that the patient has dry skin in the ear canal. Which preparation is FDA approved for this use? a. Carbamide peroxide b. Hydrogen peroxide c. Liquid docusate sodium d. Mineral oil

ANS: A Carbamide peroxide Any preparation with carbamide peroxide is FDA approved as a cerumenolytic. Patients with dry skin in the ear canal should not use any product containing hydrogen peroxide. --Liquid docusate sodium and mineral oil are often used, but do not have specific FDA approval.

A child has recurrent impaction of cerumen in both ears and the parent asks what can be done to help prevent this. What will the provider recommend? a. Clean the outer ear and canal with a soft cloth b. Removing cerumen with a cotton-tipped swab c. Try thermal-auricular therapy when needed d. Use an oral irrigation tool to remove cerumen

ANS: A Clean the outer ear and canal with a soft cloth Parents should be instructed to use a soft cloth to clean the outer ear and canal only. --Use of a cotton-tipped swab or any other implement may push cerumen deeper into the canal and risk damaging the tympanic membrane. --Thermal-auricular therapy is not recommended. --Oral irrigation tools have high pressure and risk of damage to the tympanic membrane.

Which physical examination finding suggests viral rather than bacterial parotitis? a. Clear discharge from Stensen's duct b. Enlargement and pain of affected glands c. Gradual reduction in saliva production d. Unilateral edema of parotid glands

ANS: A Clear discharge from Stensen's duct Viral parotitis generally produces clear discharge. --Enlargement and pain of affected glands may be non-specific or is associated with TB infection. --A gradual reduction in saliva, resulting in xerostomia, is characteristic of HIV infection. --Unilateral edema is more often bacterial.

A patient who has an inflamed pterygia lesion has been using loteprednol topical steroid drops for 7 days. The patient shows no improvement in symptoms. What is the next course of action? a. Consult with an ophthalmologist b. Continue the medication for 7 more days c. Prescribe a systemic corticosteroid d. Refer the patient to the emergency department

ANS: A Consult with an ophthalmologist Topical steroid medications are used to treat pterygia, but should not be used longer than 7 days without ophthalmic consultation. --Systemic corticosteroids are not indicated and an emergent referral is not necessary.

A patient has chronic chest pain that occurs after meals and the provider suspects GERD. The provider prescribes a PPI and after 2 months the patient reports improvement in symptoms. What is the next action in treating this patient? a. Continue the proton pump inhibitor b. Order esophageal pH monitoring c. Refer the patient to a gastroenterologist d. Schedule an upper endoscopy

ANS: A Continue the proton pump inhibitor Often the effectiveness of treatment with a PPI is diagnostic in itself and is equal to or better than more invasive and expensive testing. --As long as the patient continues to show improvement, there is no need to order tests or refer for evaluation.

A school-age child has had 5 episodes of tonsillitis in the past year and 2 episodes the previous year. The child's parent asks the provider if the child needs a tonsillectomy. What will the provider tell this parent? a. Current recommendations do not support tonsillectomy for this child. b. If there is one more episode in the next 6 months, a tonsillectomy is necessary. c. The child should have radiographic studies to evaluate the need for tonsillectomy. d. Tonsillectomy is recommended based on this child's history.

ANS: A Current recommendations do not support tonsillectomy for this child. Recommendations suggest 6 to 7 documented episodes of GAS within 1 year, 5/year for 2 consecutive years, or 3/year for 3 years. --Radiographic studies are not indicated.

Routine screening blood tests at an annual physical exam reveal a fasting glucose level of 125 mg/dL and a hemoglobin A1C of 6.2%. What will the provider do, based on these results? a. Evaluate the patient for impaired glucose tolerance b. Reassure the patient that these are normal values c. Suggest that the patient begin an exercise program d. Tell the patient that these results indicate diabetes

ANS: A Evaluate the patient for impaired glucose tolerance The fasting blood glucose level is normal, but the HgA1C indicates impaired glucose tolerance. --If the HgA1C were greater than 6.4%, the patient would be diagnosed with DM. --Until the results are evaluated, suggestions for treatment are not indicated.

An adult patient reports frequent episodes of syncope and lightheadedness. The provider notes a HR of 70 beats per minute. What will the provider do next? a. Evaluate the patient's orthostatic vital signs b. Monitor the patient's heart rate while the patient is bearing down c. Order an electrocardiogram and exercise stress test d. Reassure the patient that the symptoms are non-cardiac in origin

ANS: A Evaluate the patient's orthostatic vital signs Orthostatic vital signs are helpful to exclude orthostatic hypotension as a cause of syncope and is easily performed in the clinic. Assessment for vagal bradycardia may be performed next. --ECG and ETT are not recommended as an initial evaluation in a healthy patient unless other causes are not determined. --Without an assessment of the cause of the syncope, cardiac causes cannot be excluded.

A patient has been taking amoxicillin for treatment of a dental abscess. In a follow-up visit, the provider notes edema of the eyelids and conjunctivae. What is the next action? a. Hospitalize the patient for an endodontist consultation b. Prescribe amoxicillin clavulanate for 10 to 14 days c. Recommend follow up with a dentist in 2 to 3 days d. Suggest using warm compresses to the eyes for comfort

ANS: A Hospitalize the patient for an endodontist consultation This patient has signs of complications and requires hospitalization with management by a dentist or endodontist. --Changing the antibiotic without consultation is not recommended. --Prompt hospitalization is required.

A patient has parotitis and cultures are positive for actinomycosis. What is the initial treatment for this condition? a. IV penicillin b. Oral clindamycin c. Oral erythromycin d. Topical antibiotics

ANS: A IV penicillin IV penicillin, followed by oral PCN for several months is indicated for actinomycosis. --Clindamycin and erythromycin are used for PCN allergy. --Topical antibiotics are not effective.

A patient reports coughing up a small amount of blood after a week of cough and fever. The patient has been previously healthy and does not smoke or work around pollutants or irritants. What will the provider suspect as the most likely cause of this patient's symptoms? a. Infection b. Lung abscess c. Malignancy d. Thromboembolism

ANS: A Infection In a healthy patient without risk factors who has a cough and fever, infection is the most likely cause. --Lung abscess may occur, but is less likely. --Malignancy is also less likely. --Thromboembolism is more likely after surgery or with trauma

A child sustains an ocular injury in which a shard of glass from a bottle penetrated into the eyewall. The emergency department provider notes that the shard has remained in the eye. Which best describes this type of injury? a. Intraocular foreign body b. Penetrating eye injury c. Perforating eye injury d. Ruptured globe injury

ANS: A Intraocular foreign body When a portion of the insulting object enters and remains in the eye = intraocular foreign body. --penetrating = something penetrates through the eyewall without an exit wound. --perforating = object has both an entry and an exit wound. --ruptured globe = blunt force causes the eyewall to rupture.

A patient with COPD reports daily symptoms of dyspnea and cough. Which medication will the prescriber order? a. Ipratropium bromide b. Pirbuterol acetate c. Salmeterol xinafoate d. Theophylline

ANS: A Ipratropium bromide 1st) Ipratropium bromide is an anticholinergic medication and is used as first-line therapy in patients with daily symptoms. --2nd)Pirbuterol acetate and salmeterol xinafoate are both beta2-adrenergics and are used to relieve bronchospasm; pirbuterol is a short-term medication used for symptomatic relief and salmeterol is a long-term medication useful for reducing nocturnal symptoms. --3rd) Theophylline is a third-line agent.

A patient with hemoptysis and no other symptoms has a normal chest radiograph, CT, and fiberoptic bronchoscopy studies. What is the next action in managing this patient? a. Observation b. Prophylactic antibiotics c. Specialist consultation d. Surgical intervention

ANS: A Observation Patients with negative findings on CXR, CT, and bronchoscopy, with no risk factors, may be observed for 3 years. --Abx are not indicated, since signs of infection are not present. --Specialty consultation and surgery are not indicated.

During a routine health maintenance examination, the provider auscultates a cervical bruit. The patient denies syncope, weakness, or headache. What will the provider do, based on this finding? a. Order a carotid duplex ultrasound b. Order catheter-based angiography c. Refer the patient to a neurosurgeon d. Schedule a computed tomography angiography

ANS: A Order a carotid duplex ultrasound Carotid duplex ultrasound is the primary diagnostic tool for carotid stenosis. A cervical bruit in an asymptomatic patient is an indication for this test. --Catheter-based angiography is the criterion-based standard, but has inherent costs and risks. --A neurosurgery referral is not indicated without further testing. --CTA is used instead of duplex US if the test is not available, if US results are inconclusive, or further evaluation is needed based on US results.

A 75-year-old patient reports pain and a feeling of tiredness in both legs that only relieves after sitting for 30 minutes or more. What the does provider suspect as the cause for these symptoms? a. Buerger's disease b. Cauda equina syndrome c. Diabetic neuropathy d. Peripheral arterial disease

ANS: B Cauda equina syndrome Patients with cauda equina syndrome, which causes spinal stenosis, will often not get relief until they sit down for a period of time. --Buerger's disease involves both the upper and lower extremities. --Diabetic neuropathy may mask pain. --PAD involves these symptoms that stop with rest.

1. Which patient should have PFT as part of the presurgical exam? a. A patient older than 60 years of age b. A patient undergoing major intrathoracic surgery c. A patient with a history of pneumonia in the last 2 years d. A patient with diabetes and morbid obesity

ANS: B A patient undergoing major intrathoracic surgery Any patient undergoing major thoracic surgery should have pulmonary function testing. --Age over 60 years, a history of pneumonia, and DM and obesity do not require pulmonary function testing unless there is comorbid COPD.

A 60-year-old patient with a previous history of shingles asks about the herpes zoster vaccine. What will the provider recommend? a. A series of two herpes zoster vaccinations b. A single dose of herpes zoster vaccine c. No herpes zoster vaccine is necessary d. Prophylactic vaccination if exposed to chicken pox

ANS: B A single dose of herpes zoster vaccine All patients 50 years and older should have a single dose of herpes zoster vaccine regardless of previous herpes zoster infection.

A patient develops a dry, non-productive cough and is diagnosed with bronchitis. Several days later, the cough becomes productive with mucoid sputum. What may be prescribed to help with symptoms? a. Antibiotic therapy b. Antitussive medication c. Bronchodilator treatment d. Mucokinetic agents

ANS: B Antitussive medication Antitussive medications are occasionally useful for short-term relief of coughing. --Antibiotic therapy is generally not needed and should be avoided unless a bacterial cause is likely. --Bronchodilator medications show no demonstrated reduction in symptoms and are not recommended. --Mucokinetic agents have no evidence to support their use.

patient has bilateral bleeding from the nose with bleeding into the pharynx. What is the initial intervention for this patient? a. Apply firm, continuous pressure to the nostrils b. Assess airway safety and vital signs c. Clear the blood with suction to identify site of bleeding d. Have the patient sit up straight and tilt the head forward

ANS: B Assess airway safety and vital signs Bilateral epistaxis into the pharynx is more indicative of a posterior bleed which is more likely to be severe. The most important intervention is to ensure airway safety and determine the stability of vital signs. --Other measures are taken as needed.

A patient reports sustained, irregular heart palpitations. What is the most likely cause of these symptoms? a. Anemia b. Atrial fibrillation c. Extrasystole d. Paroxysmal attacks

ANS: B Atrial fibrillation Atrial fibrillation causes palpitations that are irregular and tend to be sustained. --Anemia will cause rapid palpitations that are regular. --Extrasystole causes palpitations or awareness of isolated extra beats with a pause. --Paroxysmal attacks start and terminate abruptly and are usually rapid and regular.

An older patient with COPD is experiencing dyspnea and has an oxygen saturation of 89% on room air. The patient has no history of pulmonary hypertension or congestive heart failure. What will the provider order to help manage this patient's dyspnea? a. Anxiolytic drugs b. Breathing exercises c. Opioid medications d. Supplemental oxygen

ANS: B Breathing exercises Formal pulmonary rehabilitation programs, including breathing exercises, are used to manage long-term disease such as COPD. --Anxiolytics and opioids must be used cautiously because of respiratory depression side effects. Medicare does not approve oxygen supplementation unless saturations are less than 88% on room air or for patients who have pulmonary hypertension or CHF who have saturations 89%.REF: Management

A young adult patient without a previous history of lung disease has an increased respiratory rate and reports a feeling of "not getting enough air." The provider auscultates clear breath sounds and notes no signs of increased respiratory effort. Which diagnostic test will the provider perform initially? a. Chest radiograph b. CBC c. Computerized tomography d. Spirometry

ANS: B CBC This patient has no signs indicating lung disease, but does exhibit signs of hypoxia. A CBC would evaluate for anemia, which is a more common cause of hypoxia in otherwise healthy adults. --CXR is used to evaluate infectious causes. --CT is used if interstitial lung disease is suspected. --Spirometry is useful to diagnose asthma and COPD.

A patient who has been diagnosed with heart failure for over a year reports being comfortable while at rest and experiences palpitations and dyspnea when walking to the bathroom. Which classification of heart failure is appropriate based on these symptoms? a. Class I b. Class II c. Class III d. Class IV

ANS: B Class II Patients with Class II heart failure (HF) will have slight limitation of activity and will be comfortable at rest with symptoms occurring with ordinary physical activity. --Patients with Class I HF do not have limitations and ordinary physical activity does not produce symptoms. --With Class III HF, less than usual activity will produce symptoms. --With Class IV HF, symptoms are present even at rest and all physical activity worsens symptoms.

A patient with a smoking history of 35 pack years reports having a chronic cough with recent symptoms of pink, frothy blood on a tissue. The chest radiograph shows a possible nodule in the right upper lobe. Which diagnostic test is indicated? a. Coagulation studies b. Computerized tomography c. Fiberoptic bronchoscopy d. Needle biopsy

ANS: B Computerized tomography CT is suggested for initial evaluation of patients at high risk of malignancy, (ie a smoker with >30 pack yrs), who have suspicious findings on CXR. --Coagulation studies are performed for patients taking anticoagulants or a history of coagulopathy. --Fiberoptic bronchoscopy is used with CT but is not the initial test. --Needle biopsy is performed if other tests indicate a tumor.

A young child has a pale, whitish discoloration behind the TM. The provider notes no scarring on the TM and no retraction of the pars flaccida. The parent states that the child has never had an ear infection. What do these findings most likely represent? a. Chronic cholesteatoma b. Congenital cholesteatoma c. Primary acquired cholesteatoma d. Secondary acquired cholesteatoma

ANS: B Congenital cholesteatoma Patients without a history of otitis media or perforation of the TM most likely have congenital cholesteatoma. --Primary acquired cholesteatoma will include retraction of the pars flaccida. --Secondary acquired cholesteatoma has findings associated with the underlying etiology.

A patient who has chronic lower back pain reports increased difficulty sleeping unrelated to discomfort, along with a desire to quit working. What will the provider do? a. Ask the patient about addiction issues. b. Consult with a social worker. c. Increase the dosage of prescribed pain medications. d. Order radiographic studies of the lower spine.

ANS: B Consult with a social worker. Patients who exhibit poor sleep and poor coping may be developing mental defeat as a result of chronic pain and should be evaluated and treated early for this to prevent further disability and improve functionality. Substance abuse may be a part of mental defeat and should be evaluated based on assessment findings. Unless the symptoms are related to pain, increasing the dose of analgesics and ordering diagnostic studies are not indicated.

An elderly female without prior history of cardiovascular disease reports lower leg soreness and fatigue when shopping or walking in the neighborhood. The primary care provider notes decreased pedal pulses bilaterally. Which test will the provider order initially to evaluate for peripheral arterial disease based on these symptoms? a. Digital subtraction angiography b. Doppler ankle, arm index c. Magnetic resonance angiography d. Segmental limb pressure measurement

ANS: B Doppler ankle, arm index The Doppler study may be performed easily to indicate the likelihood of PAD. --Other tests are performed only if indicated.

A patient has a chronic swelling of the parotid gland that is unresponsive to antibiotics and which has not increased in size. Which diagnostic test is indicated? a. Computed tomography b. Fine needle aspiration c. Magnetic resonance imaging d. Plain film radiography

ANS: B Fine needle aspiration Chronic lesions may represent TB or malignancies, fine needle aspiration is indicated to rule out. --Radiological studies are used to identify the extent of disease but are usually not diagnostic.

A patient reports recurrent chest pain that occurs regardless of activity and is not relieved by rest. The provider administers a nitroglycerin tablet which does not relieve the discomfort. What is the next action? a. Administer a second nitroglycerin tablet b. Give the patient a beta-blocker medication c. Prescribe a calcium channel blocker mediation d. Start aspirin therapy and refer the patient to a cardiologist

ANS: B Give the patient a beta-blocker medication Patient with these symptoms who do not respond to nitroglycerin is likely to have microvascular angina. Treatment is effective with beta-blockers. --These symptoms are not characteristic of acute MI, so aspirin is not given. --A second nitroglycerin tablet is used for classic angina. --CCB are not indicated.

A child is hit with a baseball bat during a game and sustains an injury to the nose, along with a transient loss of consciousness. A health care provider at the game notes bleeding from the child's nose and displacement of the septum. What is the most important intervention at this time? a. Apply ice to the injured site to prevent airway occlusion b. Immobilize the child's head and neck and call 911 c. Place nasal packing in both nares to stop the bleeding d. Turn the child's head to the side to prevent aspiration of blood

ANS: B Immobilize the child's head and neck and call 911 Nasal trauma resulting in loss of consciousness and possible neck injury are emergencies. The provider should take cervical spine precautions and call 911 for transport to an emergency room. --The other interventions may be performed once the child's head and neck are stable.

A patient has dacryocystitis (infection of the tear/lacrimal sac usually due to a blockage in the tear/nasolacrimal duct). The provider notes a painful lacrimal sac abscess that appears to be coming to a head. Which treatment will be useful initially? a. Eyelid scrubs with baby shampoo b. Incision and drainage c. Lacrimal bypass surgery d. Topical antibiotic ointment

ANS: B Incision and drainage When an abscess is present and coming to a head, incision and drainage may be useful. --Definitive treatment with lacrimal bypass surgery will be performed once the acute episode has resolved. --Eyelid scrubs and topical ointments are not effective.

1. During an eye examination, the provider notes a red light reflex in one eye but not the other. What is the significance of this finding? a. Normal physiologic variant b. Ocular disease requiring referral c. Potential infection in the "red" eye d. Potential vision loss in one eye

ANS: B Ocular disease requiring referral The red reflex should be elicited in normal eyes. Any asymmetry or opacity suggests ocular disease, potentially retinoblastoma, and should be evaluated immediately.

A patient has a sore throat, a temperature of 38.5° C, tonsillar exudates, and cervical lymphadenopathy. What will the provider do next to manage this patient's symptoms? a. Order an antistreptolysin O titer b. Perform a rapid antigen detection test c. Prescribe empiric penicillin d. Refer to an otolaryngologist

ANS: B Perform a rapid antigen detection test The RADT is performed initially to determine whether GAS (group A step) is present. --The ASO titer is not used during initial diagnostic screening. --Penicillin should not be given empirically. --A referral to a specialist is not required for GAS infection.

A patient reports tooth pain in a lower molar and the provider notes a mobile tooth with erythema and edema of the surrounding tissues without discharge. Which is the initial course of action by the provider? a. Perform an incision and drainage of the edematous tissue b. Prescribe amoxicillin and refer to a dentist in 2 to 3 days c. Recommend oral antiseptic rinses and follow up in one week d. Refer to an oral surgeon for emergency surgery

ANS: B Prescribe amoxicillin and refer to a dentist in 2 to 3 days The primary provider may prescribe antibiotics, especially if the surrounding tissues are infected. Patients should follow up with a dentist in 2 to 3 days. --The primary provider generally does not perform I&D; this should be done by the dentist. --Follow up should be with a dentist in 2 to 3 days, not one week. --Emergency surgery is indicated if there is a question of airway compromise.

A patient who has a history of working around asbestos and silica fibers is concerned about developing lung disease. The primary care provider determines that the patient has a previous history of asthma as a child and currently has frequent episodes of bronchitis. A PE is normal and PFTs and are nradiographs egative. What action is correct? a. Reassure the patient about the normal findings. b. Refer the patient to an occupational health specialist. c. Request a workplace environmental assessment. d. Suggest that the patient follow up with a pulmonologist.

ANS: B Refer the patient to an occupational health specialist. Patients with environmental exposure may not have symptoms or positive findings. Because this patient reports frequent bronchitis, this should be followed up with an occupational health specialist who can evaluate the degree of exposure and perform further testing. Normal findings are not necessarily reassuring. The occupational specialist may request an environmental assessment. Pulmonologists are not trained in occupational health.

A patient reports painful swelling in the mouth with increased pain at mealtimes. The provider notes a mass in the salivary gland region. What is the likely cause of these symptoms? a. Basal cell adenoma b. Sialolithiasis c. Sjogren syndrome d. Warthin's tumor

ANS: B Sialolithiasis a non-infectious salivary gland disorder characterized by pain at mealtimes caused by blockage of the salivary duct by stones. --Basal cell adenoma is a noninfectious cause of salivary gland inflammation that is generally painless. --Sjogren syndrome manifests with xerostomia and abnormal taste. --Warthin's tumor causes a painless, unilateral mass.

An adult patient has epiglottitis secondary to a chemical burn. Which medication will be given initially to prevent complications? a. Chloramphenicol b. Clindamycin c. Dexamethasone d. Metronidazole

ANS: C Dexamethasone This case of epiglottitis does not have an infectious cause, so abx are not given unless there are symptoms of infection. --A corticosteroid can decrease the need for intubation.

Which test is diagnostic for diagnosing myocarditis? a. Echocardiogram b. Electrocardiogram c. Endomyocardial biopsy d. Magnetic resonance imaging

ANS: C Endomyocardial biopsy the only definitive test to diagnose myocarditis. -Other tests are useful in determining symptoms but are not specific to this diagnosis.

A patient comes to an emergency department with chest pain. The patient describes the pain is sharp and stabbing and reports that it has been present for several weeks. Upon questioning, the examiner determines that the pain is worse after eating. The patient reports getting relief after taking a friend's nitroglycerin during one episode. What is the most likely cause of this chest pain? a. Aortic dissection pain b. Cardiac pain c. Esophageal pain d. Pleural pain

ANS: C Esophageal pain Pain that is constant for weeks or is sharp and stabbing is not likely to be cardiac in origin. Both esophageal and cardiac causes will be attenuated with sublingual nitroglycerin. --Aortic dissection will cause an abrupt onset with the greatest intensity at the beginning of the pain. --Pleural pain is usually related to deep breathing or cough.

An adult patient is seen in clinic with fever, sore throat, and dysphagia. Which diagnostic test will the provider order to confirm a diagnosis of epiglottitis? a. Blood cultures b. Complete blood count c. Fiberoptic nasopharyngoscopy d. Lateral neck film

ANS: C Fiberoptic nasopharyngoscopy allows direct visualization of the epiglottis and is used increasingly with adult patients suspected of having epiglottitis. --Blood cultures and a CBC may be drawn as part of the workup to help guide antimicrobial therapy, but are not diagnostic. --A lateral neck film is not always diagnostic with adults.

A patient who is a runner is diagnosed with viral myocarditis and asks when he may begin exercising again. What will the provider tell this patient? a. Exercise is contraindicated for life b. Exercise may resume when symptoms subside c. He may resume exercise in 6 months d. He must be symptom-free for 1 year

ANS: C He may resume exercise in 6 months Patients with myocarditis should not exercise for 6 months after the onset of symptoms.

A patient reports painful oral lesions 3 days after feeling pain and tingling in the mouth. The provider notes vesicles and ulcerative lesions on the buccal mucosa. What is the most likely cause of these symptoms? a. Bacterial infection b. Candida albicans c. Herpes simplex virus d. Human papilloma virus

ANS: C Herpes simplex virus HSV infections generally start with a prodrome of tingling, pain, and burning followed by vesicular and ulcerative lesions. --Bacterial infection presents with inflammation of the gingiva, bleeding, and ulceration with or without purulent discharge. --Candida albicans appear as white, cottage cheese-like lesions that may be removed, but may cause bleeding when removed. --HPV manifests as white, verrucous lesions individually or in clusters.

2. A patient has a gradually enlarging nodule on one upper eyelid and reports that the lesion is painful. On examination, the lesion appears warm and erythematous. The provider knows that this is likely to be which type of lesion? a. Blepharitis b. Chalazion c. Hordeolum d. Meibomian

ANS: C Hordeolum (stye) Although hordeolum and chalazion lesions both present as gradually enlarging nodules, a hordeolum is usually painful, while a chalazion generally is not. ---Blepharitis refers to generalized inflammation of the eyelids. --Meibomian is a type of gland near the eye.

A patient is brought to an emergency department with symptoms of STEMI. The nearest hospital that can perform percutaneous coronary intervention (PCI) is 3 hours away. What is the initial treatment for this patient? a. Administer heparin b. Give the patient an oral beta blocker c. Initiate fibrinolytic treatment d. Transfer to the PCI-capable institution

ANS: C Initiate fibrinolytic treatment Fibrinolytic therapy should be administered to any patient with evolving STEMI within 30 minutes of the time of first medical contact. --Patients more than 120 minutes away from a PCI-capable hospital should be given fibrinolytic therapy since PCI should be performed within 90 minutes if possible. --Giving heparin or beta blockers is not helpful.

An elderly patient who has orthostatic hypotension secondary to antihypertensive medications is noted to have a drop in systolic blood pressure of 25 mm Hg. Which intervention is important for this patient? a. Administration of intravenous fluids b. Close monitoring cardiorespiratory status c. Initiation of a fall risk protocol d. Withholding antihypertensive medications

ANS: C Initiation of a fall risk protocol A reduction of systolic blood pressure >20 mm Hg is a risk factor for falls in the elderly, so a fall risk protocol should be initiated. --Unless the patient is dehydrated, IV fluids are not recommended. --Close monitoring of CR status will not prevent falls. --Withholding antihypertensive medications often worsens orthostatic hypotension.

Which is the most common cause of orbital cellulitis in all age groups? a. Bacteremic spread from remote infections b. Inoculation from local trauma or bug bites c. Local spread from the ethmoid sinus d. Paranasal sinus inoculation

ANS: C Local spread from the ethmoid sinus The membrane separating the ethmoid sinus from the orbit is paper-thin and is the most common source of orbital infection in all age groups. --Bacteremic spread, inoculation from localized trauma, and paranasal sinus spread all may occur but are less common.

A 60-year-old patient who leads a sedentary lifestyle has expressed an interest in beginning an aerobic exercise program. What will the provider include when counseling this patient about this program? a. Begin with a 45 to 60 minute workout b. Include a 1 to 2 minute warm up before exercise c. Maintain a heart rate between 80 and 128 beats per minute d. Stretching should be performed prior to activity

ANS: C Maintain a heart rate between 80 and 128 beats per minute The heart rate should be kept between 50% and 80% of the maximum heart rate (220 minus the patient's age = 160), which is 80 to 128 beats per minute. --Patients who are not conditioned should begin with a 20 minute workout; conditioned individuals may increase up to 60 minutes. --The warm up should be 3 to 5 minutes and longer if it is cold. --Stretching is performed after the activity when the muscles are warm.

1. A patient returns home from travel in Africa and experiences chronic, non-bloody diarrhea. The patient reports frequent bloating and flatulence with a "rotten egg" smell. What is the treatment for this type of diarrhea? a. Azithromycin b. Ciprofloxacin c. Metronidazole d. Rifampin

ANS: C Metronidazole This patient has symptoms characteristic of Giardia lamblia and should be treated with metronidazole. --Azithromycin is given for Campylobacter infection, which is a bacterial cause. --Ciprofloxacin is used for bacterial infections such as Salmonella or Shigella, which cause bloody diarrhea. --Rifampin is used for non-invasive strains of E. coli.

Which is characteristic of chronic bronchitis and not emphysema? a. Damage to the alveolar wall b. Destruction of alveolar architecture c.Mild alteration in lung tissue compliance d. Mismatch of ventilation and perfusion

ANS: C Mild alteration in lung tissue compliance Chronic bronchitis causes much less parenchymal damage than emphysema does, so there is a milder alteration in lung tissue compliance in chronic bronchitis. -The other symptoms are characteristic of emphysema.

A patient is in the emergency department with unilateral epistaxis that continues to bleed after 15 minutes of pressure on the anterior septum and application of a topical nasal decongestant. The provider is unable to visualize the site of the bleeding. What is the next measure for this patient? a. Chemical cautery b. Electrocautery c. Nasal packing d. Petrolatum ointment

ANS: C Nasal packing Nasal packing is used if bleeding continues after initial measures. --Chemical cautery and electrocautery are used only if the site of bleeding is visualized. --Petrolatum ointment is applied once the bleeding is stopped.

The provider sees a child with a history of high fever and sore throat. When entering the exam room, the provider finds the child sitting in the tripod position and notes stridor, drooling, and anxiety. What is the initial action for this patient? a. Administer empiric intravenous antibiotics and steroids b. Have the child lie down and administer high-flow, humidified oxygen c. Obtain an immediate consultation with an otolaryngologist d. Perform a thorough examination of the oropharynx

ANS: C Obtain an immediate consultation with an otolaryngologist Patients with suspected epiglottitis, with high fever, sore throat, stridor, drooling, and respiratory distress, should be referred immediately to otolaryngology. Starting an IV or having the child lie down will increase distress and may precipitate laryngospasm. The throat should not be examined because it may cause laryngospasm REF: Diagnostics/Management

A patient has native valve endocarditis. While blood cultures are pending, which antibiotics will be ordered as empiric treatment? a. A beta-lactamase resistant penicillin and an antifungal drug b. Imipenem-cilastin and ampicillin c. Penicillin and an aminoglycoside antibiotic d. Vancomycin and quinupristin-dalfopristin

ANS: C PCN & an aminoglycoside abx The most common organism in NVE is PCN & an aminoglycoside is needed, although most strains causing NVE are not PCN-resistant. --Antifungal infections are rare and antifungal medications are not part of empiric therapy. --Imipenem-cilcastin plus ampicillin is given for identified Enterococcus faecalis infection. --Vancomycin and quinupristin-dalfopristin is used, with limited evidence for benefit, for Enterococcus faecium infection.

A patient has gingival inflammation with several areas of ulceration and a small amount of purulent discharge. What is required to diagnose this condition? a. Culture and sensitivity b. Microscopic exam of oral scrapings c. Physical examination d. Tzanck smear

ANS: C Physical examination This patient has symptoms consistent with gingivitis may be diagnosed by physical examination alone. --Cultures are not necessary unless systemic disease is present. --A microscopic exam of oral scrapings to look for hyphae may be performed to diagnose candida infections. --A Tzanck smear is performed to confirm a diagnosis of herpes simplex.

A patient reports ear pain and difficulty hearing. An otoscopic examination reveals a small tear in the tympanic membrane of the affected ear with purulent discharge. What is the initial treatment for this patient? a. Insert a wick into the ear canal b. Irrigate the ear canal to remove the discharge c. Prescribe antibiotic ear drops d. Refer the patient to an otolaryngologist

ANS: C Prescribe antibiotic ear drops This is most likely due to infection and should be treated with antibiotic ear drops. --Wicks are used for otitis externa. --The ear canal should not be irrigated to avoid introducing fluid into the middle ear. --It is not necessary to refer unless the perforation does not heal.

A patient who has Class II heart failure is taking an ACE inhibitor and reports a recurrent cough that does not interfere with sleep or activity. What will the provider do initially to manage this patient? a. Assess serum potassium and sodium immediately b. Discontinue the ACE inhibitor and prescribe an ARB c. Provide reassurance that this is a benign side effect d. Withhold the drug and evaluate renal and pulmonary function

ANS: C Provide reassurance that this is a benign side effect Cough occurs in about 20% of patients who take ACE inhibitors and is not dangerous. The patient should be reassured that this is the case. --If the cough is annoying, alternate therapy with an ARB may be considered. --It is not necessary to evaluate electrolytes, renal fxn, or pulmonary function.

2. A patient comes to the clinic with diffuse erythema (reddening) in one eye without pain or history of trauma. The examination reveals a deep red, confluent hemorrhage in the conjunctiva of that eye. What is the most likely treatment for this condition? a. Order lubricating drops or ointments b. Prescribe ophthalmic antibiotic drops c. Reassure the patient that this will resolve d. Refer to an ophthalmologist

ANS: C Reassure the patient that this will resolve Most subconjunctival hemorrhage, occurring with trauma or Valsalva maneuvers, will self-resolve and are benign. --Lubricating drops are used for chemosis. --Antibiotic eye drops are not indicated. --Referral is not indicated.

A high school athlete reports recent onset of chest pain that is aggravated by deep breathing and lifting. A 12-lead electrocardiogram in the clinic is normal. The examiner notes localized pain near the sternum that increases with pressure. What will the provider do next? a. Order a chest radiograph b. Prescribe an antibiotic c. Recommend an NSAID d. Refer to a cardiologist

ANS: C Recommend an NSAID This patient has symptoms consistent with chest wall pain because the chest pain occurs with specific movement and is easily localized. --Since the ECG is normal, there is no need to refer to a cardiologist. --The patient does not have symptoms of pneumonia so a radiograph or abx is not needed. --NSAIDs are recommended for comfort

2. A high-school adolescent is being screened for fitness before participating in sports. The adolescent has a normal examination and the examiner notes S1 and S2 heart sounds without murmur, normal blood pressure, and equal pulses. The parent reports that the adolescent's father has a history or Wolff-Parkinson-White syndrome, which has been treated. What will the provider do? a. Clear the adolescent to play sports. b. Perform an electrocardiogram. c. Refer the adolescent to a cardiologist. d. Tell the adolescent that sports are not allowed.

ANS: C Refer the adolescent to a cardiologist. A positive family history of Wolff-Parkinson-White syndrome requires physician consultation or referral before medical clearance can be given. The adolescent has a normal heart rate and physical exam, so the ECG may not yield significant or useful results. The examiner cannot clear the adolescent without consulting with a specialist. The adolescent may be cleared for sports by the specialist.

A patient reports heart palpitations but no other symptoms and has no prior history of cardiovascular disease. The clinic provider performs an electrocardiogram and notes atrial fibrillation and a heart rate of 120 beats per minute. Which is the initial course of action in treating this patient? a. Administer atenolol intravenously b. Admit to the hospital for urgent cardioversion c. Refer the patient to a cardiologist d. Transport the patient to the ED by ambulance

ANS: C Refer the patient to a cardiologist This patient has no history of serious heart disease and does not have symptoms of chest pressure, acute MI, or CHF and may be referred to a cardiologist for evaluation and treatment. --Atenolol is given IV for patients who are unstable; the advanced life support treatment guidelines do not recommend treatment of tachycardia if the patient is stable. --Urgent cardioversion is rarely needed if the heart rate is less than 150 beats per minute unless there are underlying heart conditions. --It is not necessary to transport a stable patient to the ED.

3. An overweight adolescent who takes metformin has type 2 diabetes with a HgA1C of 8.5% and asks about sports participation. What will the provider recommend? a. Losing weight prior to initiating sports participation b. Participation in strenuous sports to help with weight loss c. Referral to the endocrinologist for sports clearance d. Switching to insulin therapy prior to participation

ANS: C Referral to the endocrinologist for sports clearance Patients with poorly controlled diabetes should be referred to a specialist prior to clearance for sports participation. This patient has an elevated HgA1C, indicating poor control. The endocrinologist may suggest the other options, but the PCP should not clear this patient for participation in sports.

2. A patient who is planning international travel to a developing country asks the provider about vaccinations. Which is true about pretravel vaccines? a. Country-specific guidelines are provided by individual embassies. b. Malaria vaccine is the most important vaccine for worldwide travel. c. Requirements should be reviewed at least 4 to 6 weeks prior to travel. d. There are at least five required vaccines for entry into certain countries.

ANS: C Requirements should be reviewed at least 4 to 6 weeks prior to travel so that antibody responses and completion of vaccine series may occur. --Country-specific guidelines may be found on the CDC website. --Malaria is not prevented by vaccine, but by prophylactic antimalarial drugs. --There are only 2 required vaccines.

A patient with chronic leg pain describes the pain as "stabbing" and "throbbing." This is characteristic of which type of pain? a. Neuropathic pain b. Referred pain c. Somatic pain d. Visceral pain

ANS: C Somatic pain Somatic pain is caused by the activation of nociceptors in the peripheral tissues, including skin, bones, muscles, and soft tissue and is usually well-localized and characterized as stabbing, aching, or throbbing. Neuropathic pain occurs from injury to or disease of the nervous system and is described as burning, shooting, or tingling. Referred pain is a kind of visceral pain that is localized, but not attributable to the involved organ. Visceral pain is related to an organ and is often referred and poorly localized.

A patient is suspected of having vestibular neuritis. Which finding on physical examination is consistent with this diagnosis? a. Facial palsy and vertigo b. Fluctuating hearing loss and tinnitus c. Spontaneous horizontal nystagmus d. Vertigo with changes in head position

ANS: C Spontaneous horizontal nystagmus spontaneous horizontal or rotary nystagmus, away from the affected ear. --Facial palsy with vertigo = Ramsay Hunt syndrome, caused by herpes zoster. --Fluctuating hearing loss with tinnitus = Meniere's dz. --Tinnitus may occur with vestibular neuritis, but no hearing loss. --Those with benign paroxysmal positional vertigo will exhibit vertigo with changes in head position.

A patient who has acute suppurative parotitis has been taking amoxicillin-clavulanate for 4 days without improvement in symptoms. The provider will order an antibiotic for Methicillin-resistant S. aureus. Which other measure may be helpful? a. Cool compresses b. Discouraging chewing gum c. Surgical drainage d. Topical corticosteroids

ANS: C Surgical drainage If improvement does not occur after 3 to 4 days of antibiotics, surgical drainage is appropriate. Warm compresses are recommended for comfort. Chewing gum and other sialogogues to stimulate the production of saliva are recommended. --Steroids are questionable and topical steroids will have little effect.

The AHA recommends early CPR and AED use for adult victims of cardiac arrest outside of a hospital setting because most victims have which arrhythmia? a. Atrial fibrillation b. Atrial flutter c. Ventricular fibrillation d. Ventricular tachycardia

ANS: C Ventricular fibrillation Most victims of cardiac arrest are in ventricular fibrillation, so the AHA considers early defibrillation the most effective treatment for adult victims of cardiac arrest. --The other arrhythmias are not usually present in cardiac arrest and are treated with synchronized cardioversion

3. A patient is beginning treatment for chronic pain and is unable to tolerate nonsteroidal anti-inflammatory drugs. What will the provider prescribe for this patient? a. A mixed opiate product b. A pure opioid compound c. A referral for a nerve block procedure d. A selective serotonin reuptake inhibitor (SSRI)

ANS: D A selective serotonin reuptake inhibitor (SSRI) Using the three-step analgesic ladder, the provider should use step 1 medications that include NSAIDs, tricyclic antidepressants, selective serotonin reuptake inhibitors, or anticonvulsants. Since the patient cannot tolerate NSAIDs, an SSRI is an appropriate choice. The next step if these fail is a mixed opioid product. The third step is a pure opioid product. If medication therapy fails, a referral for nerve block may be necessary.

A patient who takes a beta blocker medication is in the emergency department with syncope, shortness of breath, and hypotension. A cardiac monitor reveals a heart rate of 35 beats per minute. Which medication may be used to stabilize this patient? a. Adenosine b. Amiodarone c. Atropine d. Epinephrine

ANS: D Epinephrine Epinephrine is indicated if unstable bradycardia is caused by beta-blockers. --This patient is symptomatic and unstable and should be treated. --Adenosine and amiodarone are used to treat tachycardia. --Atropine is used for some types of bradycardia, but not when induced by beta-blockers.

An alert, irritable 12-month-old child is brought to the emergency department by a parent who reports that the child fell into a coffee table. The child has epistaxis, periorbital ecchymosis, and nasal edema. Nares are patent and the examiner palpates instability and point tenderness of the nasal septum. The orbital structures appear intact. What is an urgent action for this patient? a. Assessment of tetanus vaccination b. Ice, head elevation, and analgesia c. Immediate nasal reduction surgery d. Involvement of social services

ANS: D Involvement of social services Young children and infants generally do not engage in activities that cause the high impact needed to cause a nasal fracture and nasal structures, which have more cartilage than adults, are at much lower risk of fracture. Child abuse must be suspected in this case. --Assessment of tetanus status and application of symptomatic treatment may be ongoing, but are not urgent. --Nasal reduction surgery may be deferred for several days.

An elderly patient reports experiencing syncope each morning when getting out of bed. Which assessment will the health care provider perform first to evaluate this patient's symptoms? a. Cardiac enzyme levels b. Electroencephalogram c. Fasting blood glucose d. Orthostatic blood pressures

ANS: D Orthostatic blood pressures Orthostatic blood pressures should be measured first since this patient reports problems associated with rising from a supine position. --The other tests are performed as part of the diagnostic workup only if indicated by associated symptoms or suspected causes.

A patient who was initially treated as an outpatient for pneumonia and then hospitalized for two weeks after no improvement continues to show no improvement after several antibiotic regimens have been attempted. What is the next step in managing this patient? a. Administration of the pneumonia vaccine b. Increasing the dose of the antibiotics c. Open lung biopsy d. Performing diagnostic bronchoscopy

ANS: D Performing diagnostic bronchoscopy Patients who do not respond to antibiotic therapy may have opportunistic fungal or other infections, bronchogenic carcinoma, or other diseases. Bronchoscopy can exclude or confirm these. --The pneumonia vaccine is preventative for pneumococcal causes and will not help this patient. --Increasing the dose of the abx is not recommended. Open lung biopsy may be performed if bronchoscopy is inconclusive.

Which is an important protective precaution in a metal fabrication workshop? a. 2 mm polycarbonate safety glasses b. Eyewash stations c. Glasses with UVB protection d. Polycarbonate goggles

ANS: D Polycarbonate goggles have better side protection, will protect from foreign bodies that can reach around other lenses & should be used in very high-risk activities, such as hammering metal on metal or grinding. --2 mm polycarbonate safety glasses are a minimum safety precaution. --Glasses with UVB protection are used in occupations where sunlight exposure is high. --Eyewash stations are necessary where splash injuries or chemical exposures are possible

Which cardiac arrhythmia in an unstable patient requires unsynchronized shocks, or defibrillation? a. Atrial fibrillation b. Atrial flutter c. Monomorphic ventricular tachycardia d. Polymorphic ventricular tachycardia

ANS: D Polymorphic ventricular tachycardia Polymorphic ventricular tachycardia should be treated as ventricular fibrillation with unsynchronized shocks. --The other arrhythmias are treated with synchronized cardioversion.

A woman who is obese has a neck circumference of 16.5 cm. Which test is necessary to assess for complications of obesity in this patient based on this finding? a. Electrocardiography b. Gallbladder ultrasonography c. Mammography d. Polysomnography

ANS: D Polysomnography (sleep study) Women with a neck circumference greater than 16 cm have an increased risk of obstructive sleep apnea and should have polysomnography to assess for obstructive sleep apnea. The other tests may be necessary for obese patients but are not specific to this finding.

3. A provider performs an eye examination during a health maintenance visit and notes a difference of 0.5 mm in size between the patient's pupils. What does this finding indicate? a. A relative afferent (nerve conduction inward) pupillary defect b. Indication of a difference in intraocular pressure c. Likely underlying neurological abnormality d. Probable benign, physiologic anisocoria (difference in pupil size)

ANS: D Probable benign, physiologic anisocoria A difference in diameter of less than 1 mm is usually benign. --Afferent pupillary defects are paradoxical (inconsistent) dilations of pupils in response to light. This does not indicate differences in intraocular pressure. --A difference of more than 1 mm is more likely to represent an underlying neurological abnormality.

A patient who has heart failure with reduced ejection fraction will have which symptoms? a. Dyspnea and fatigue without volume overload b. Impairment of ventricular filling and relaxation c. Mild, exertionally related dyspnea d. Pump failure from left ventricular systolic dysfunction

ANS: D Pump failure from left ventricular systolic dysfunction HF with reduced EF results in pump failure from ventricular systolic dysfunction. --HF with preserved EF may have milder symptoms and is associated with impairment of ventricular filling and relaxation.

A patient who has a central line develops respiratory compromise. What is the initial intervention for this patient? a. Lung ultrasonography to determine the cause b. Obtaining cultures and starting antibiotics c. Prompt removal of the central line d. Rapid assessment and resuscitation

ANS: D Rapid assessment and resuscitation Patients with central lines are at increased risk for pneumothorax. Acute respiratory distress is a medical emergency and assessment and resuscitation should begin immediately. -Lung US, cultures and abx, and removal of the central line may be performed if indicated when the patient is stabilized.

A patient reports ear pain after being hit in the head with a baseball. The provider notes a perforated tympanic membrane. What is the recommended treatment? a. Order antibiotic ear drops if signs of infection occur b. Prescribe analgesics and follow up in 1 to 2 days c. Reassure the patient that this will heal without problems d. Refer the patient to an otolaryngologist for evaluation

ANS: D Refer the patient to an otolaryngologist for evaluation Patients with traumatic or blast injuries causing perforations of the tympanic membranes should be referred to specialists to determine whether damage to inner ear structures has occurred. For an uncomplicated perforation, the other interventions are all appropriate. REF: Complications and Indications for Referral

A healthy 20-year-old patient reports having had 1 or 2 episodes of syncope without loss of consciousness. Which is the most likely type of syncope in this patient? a. Cardiac b. Neurogenic c. Orthostatic hypotensive d. Reflex syncope

ANS: D Reflex syncope Neurally mediated or reflex syncope is the most common cause of syncope and is primarily seen in young adults. --Cardiac, neurogenic, and orthostatic syncope are generally seen in older adults.

A 70-year-old male patient has an aortic aneurysm measuring 5.0 cm. The patient has poorly-controlled HTN, and decompensated heart failure. What is the recommendation for treatment for this patient? a. Endovascular stent grafting of the aneurysm b. Immediate open surgical repair of the aneurysm c. No intervention is necessary for this patient d. Serial ultrasonographic surveillance of the aneurysm

ANS: D Serial ultrasonographic surveillance of the aneurysm This patient's aneurysm is <5.5 cm & repair is not necessary at this time. --Serial US surveillance is necessary to continue to evaluate size. --Repair is risky in patients with HTN & HF, so avoiding procedures if possible is recommended.

Which test is most diagnostic for COPD? a. COPD Assessment Test b. Forced expiratory time maneuver c. Lung radiograph d. Spirometry for FVC and FEV1

ANS: D Spirometry for FVC and FEV1 Spirometry testing is the gold standard for diagnosis and assessment of COPD because it is reproducible and objective. The forced expiratory time maneuver is easy to perform in a clinic setting and is a good screening to indicate a need for confirmatory spirometry. --Lung radiographs are non-specific, but may indicate hyperexpansion of lungs. --The COPD assessment test helps measure health status impairment in persons already diagnosed with COPD.

A female freshman college student tells the PCP at the student health center that she has a history of anorexia nervosa that has been well-controlled for several years. What will the provider recommend for this student? a. Dietary counseling b. Participation in sports c. Regular weight assessments d. Stress management strategies

ANS: D Stress management strategies Students with previous eating disorders may regress when stressed, so stress management is essential. Unless she begins to regress, dietary counseling is not indicated. Many who participate in sports will develop eating disorders in order to control weight. It is not necessary to evaluate weight regularly.

2. A female freshman college student tells the primary care provider at the student health center that she has a history of anorexia nervosa that has been well-controlled for several years. What will the provider recommend for this student? a. Dietary counseling b. Participation in sports c. Regular weight assessments d. Stress management strategies

ANS: D Stress management strategies Students with previous eating disorders may regress when stressed, so stress management is essential. Unless she begins to regress, dietary counseling is not indicated. Many who participate in sports will develop eating disorders to control weight. It is not necessary to evaluate weight regularly.

A patient has painful oral lesions and the provider notes several white, verrucous lesions in clusters throughout the mouth. What is the recommended treatment for this patient? a. Nystatin oral suspension b. Oral acyclovir c. Oral hygiene measures d. Surgical excision

ANS: D Surgical excision White, verrucous lesions in clusters are diagnostic for HPV infection which is treated with surgical excision. --Nystatin suspension is given for candida infection. --Oral acyclovir is used for HSV infection. --Oral hygiene measures are used for gingivitis.

1. A female college student seeks information about emergency contraception. What is the most important part of the assessment of this patient? a. Cultural considerations for use of contraception b. Feelings of guilt about a possible pregnancy c. Possible concerns about confidentiality d. The female's sense of control in sexual situations

ANS: D The female's sense of control in sexual situations Because college women are at greater risk for sexual violence and assault, a request for emergency contraception must be followed by an evaluation of possible rape or assault. The other considerations may be part of the assessment but are not as important as determining whether a rape has occurred.

What is important about increased PAI-1 levels in patients with metabolic syndrome? a. They cause increased insulin resistance. b. They predispose patients to dyslipidemia. c. They lower the risk of hypertension. d. They increase the risk of arterial thrombosis.

ANS: D They increase the risk of arterial thrombosis.

3. What is not a goal of the Healthy People initiative? a. To increase a patient's quality of life b. To create physical environments that promote proper health c. To achieve health equality by eliminating disparities d. To provide free health care to those unable to pay for care e. To promote healthy behaviors across all life stages

ANS: D To provide free health care to those unable to pay for care Overarching goals of the Healthy People initiative are to 1) increase quality and length of life, free of preventable disease, disability, injury, and premature death; 2) to achieve health equality by eliminating disparities; to create social and physical environments that promote proper health 3) to promote an increased quality of life, healthy development, and healthy behaviors across all life stages all goals consistent with the definition of wellness. --Free health care is not a stated goal of the initiative.

Which method of treatment is used for traumatic pneumothorax? a. Needle aspiration of the pneumothorax b. Observation for spontaneous resolution c. Placement of a small-bore catheter d. Tube thoracostomy

ANS: D Tube thoracostomy Traumatic pneumothorax requires tube thoracostomy because of its ability to drain larger volumes of air along with blood and fluids. --Needle aspiration is safe for primary pneumothorax. --Observation for spontaneous resolution is indicated for small pneumothoraxes.

A screening audiogram on a patient is abnormal. Which test may the primary provider perform next to further evaluate the cause of this finding? a. Impedance audiometry b. Pure tone audiogram c. Speech reception test d. Tympanogram

ANS: D Tympanogram performed to determine tympanic membrane mobility and may identify infection, fluid, or changes in middle ear pressure. --The other tests are performed by audiologists, not PCPs.

A patient reports abdominal and back pain with anorexia and nausea. During an exam, the provider notes a pulsatile abdominal mass. What is the initial action? a. Immediate referral to a thoracic surgeon b. Ordering computerized tomography angiography c. Scheduling an MRI to evaluate for aortic disease d. Ultrasound of the mass to determine size

ANS: D Ultrasound of the mass to determine size This patient has symptoms consistent with an aortic aneurysm. The initial step is to determine the size of the aneurysm; this can be done by US. --Immediate referral is not necessary. --MRI and CT diagnostic tests are ordered before surgery to evaluate the characteristics of the aneurysm.

A provider performs a nasal speculum examination on a patient who sustained nasal trauma in an MVA. The provider notes marked swelling of the nose, instability and crepitus of the nasal septum with no other facial bony abnormalities and observes a rounded bluish mass against the nasal septum. Which action is necessary at this time? a. CT scan of facial structures b. Ice packs to reduce facial swelling c. Surgery to reduce the nasal fracture d. Urgent drainage of the mass

ANS: D Urgent drainage of the mass A rounded bluish or purplish mass indicates a septal hematoma and must be drained urgently for cosmetic purposes to prevent loss of nasal cartilage caused by loss of blood supply to this area. --This patient has no signs of facial fractures, so this exam may be deferred. --Ice packs are part of ongoing management, but not a priority. --The nasal fracture may be reduced within the first 3 to 5 days after injury.

A primary care provider notes painless, hard lesions on a patient's external ears that expel a white crystalline substance when pressed. What diagnostic test is indicated? a. Biopsy of the lesions b. Endocrine studies c. Rheumatoid factor d. Uric acid chemical profile

ANS: D Uric acid chemical profile These lesions are consistent with gout and uric acid deposits. The provider should evaluate this by ordering a uric acid chemical profile. --Biopsy is indicated for any small, crusted, ulcerated, or indurated lesion that doesn't heal. --Rheumatoid nodules indicate a need for rheumatoid profiles. --Endocrine studies are ordered for patients with calcification nodules.

A child with a history of asthma is brought to the clinic with a rapid heart rate. A cardiac monitor shows a heart rate of 225 beats per minute. The provider notifies transport to take the child to the emergency department. What initial intervention may be attempted in the clinic? a. Administration of intravenous adenosine b. Giving a beta blocker c. Providing a loading dose of digoxin d. Using a vagal maneuver or carotid massage

ANS: D Using a vagal maneuver or carotid massage This child has paroxysmal supraventricular tachycardia. Vagal maneuvers or carotid massage may be attempted to slow the ventricular rate. --Adenosine is contraindicated in patients with asthma. --Medications such as beta blockers and digoxin are not used in emergency treatment of PSVT

A previously healthy patient develops myocarditis and presents with a sudden onset of dyspnea, fatigue, and orthopnea. A family history is negative. The provider suspects myocarditis. What is the most likely etiology for this patient? a. Autoimmune disorder b. Bacterial infection c. Protozoal infection d. Viral infection

ANS: D Viral infection Viral infection is the most common cause of myocarditis. --Other infections are less likely. --Although this patient may have an autoimmune disorder, the absence of family history makes this somewhat less likely.

A patient is diagnosed with PAD and elects not to have angioplasty after an angiogram reveals partial obstruction in lower extremity arteries. What will the provider recommend to help with the relief of symptoms in this patient? a. Daily aspirin therapy to prevent clotting b. Statin therapy with clopidogrel c. Walking slowly for 15 to 20 minutes twice daily d. Walking to the point of pain each day

ANS: D Walking to the point of pain each day Studies have demonstrated that an exercise program involving walking to the point of pain is as effective as angioplasty. --Medications are useful to prevent progression of plaque formation and to prevent MI.

1. Which medications are associated with weight gain? Select all that apply. Antibiotics Antidepressants Antihistamines Insulin analogs Seizure medications

Antidepressants Antihistamines Insulin analogs Seizure medications Antidepressants, antihistamines, insulin and insulin analogs, and seizure medications are all associated with weight gain. Antibiotics are not associated with weight gain.

A child has a localized nodule on one eyelid which is warm, tender, and erythematous. On examination, the provider notes clear conjunctivae and no discharge. What is the recommended treatment? a. Referral to an ophthalmologist b. Surgical incision and drainage c. Systemic antibiotics d. Warm compresses and massage of the lesion

ANS: D Warm compresses and massage of the lesion This child has a hordeolum (stye), which is generally self-limited and usually spontaneously improves with conservative treatment. Warm compresses and massage of the lesion are recommended. --A referral is not necessary unless secondary infection occurs. --Surgical intervention is not indicated. --Systemic antibiotics are used to treat secondary cellulitis.

Current American Heart Association (AHA) recommendations include: Select all that apply. a. A compression depth of 1½ inches or more on an adult b. A rate of 100 compressions per minute at a minimum c. Rescue breaths given during 2 seconds to allow full chest rise d. Untrained rescuers giving compressions without breaths e. Using a ratio of 2 rescue breaths to 30 compressions

B A rate of 100 compressions per minute at a minimum D Untrained rescuers giving compressions without breaths E Using a ratio of 2 rescue breaths to 30 compressions The AHA recommends compression rates of at least 100 compressions per minute at a ratio of 2 breaths for every 30 compressions. --Untrained rescuers are encouraged to provide chest compressions only. --The depth of compressions in adults should be at least 2 inches. --Rescue breaths are given over 1 second with full chest rise.

What are factors associated with acute suppurative parotitis? Select all that apply. a. Allergies b. Anticholinergic medications c. DM d. Hypervolemia e. Radiotherapy

B Anticholinergic medications C DM E Radiotherapy Anticholinergic medications decrease salivary flow and increase the risk for parotitis. Chronic diseases, including DM, can increase the risk. Radiotherapy and other procedures may increase the risk. --Allergies and hypervolemia do not increase the risk.

Which medications are associated with weight gain? (Select all that apply.) a. Antibiotics b. Antidepressants c. Antihistamines d. Insulin analogs e. Anticonvulsants

B Antidepressants C Antihistamines D Insulin analogs E Anticonvulsants Antidepressants, antihistamines, insulin and insulin analogs, and seizure medications are all associated with weight gain. --Antibiotics are not associated with weight gain.

Which are causes of pleural effusions? Select all that apply. a. Allergies b. Breast cancer c. Bronchiectasis d. CHF e. Dehydration

B Breast cancer C Bronchiectasis D CHF Breast cancer, bronchiectasis, and CHF can all cause pleural effusions. --Allergies and dehydration do not

A patient has eyelid swelling with erythema and warmth and reports pain with eye movement. Which diagnostic tests will be performed to confirm a diagnosis of orbital cellulitis? Select all that apply. a) Blood cultures b) CBC c) CT scan of orbits d) Lumbar puncture e) Visual acuity testing

B CBC C CT scan of orbits A CBC distinguishes infectious from non-infectious orbital cellulitis. A CT scan of the orbits confirms orbital cellulitis. --Blood cultures do not confirm, but may evaluate for septicemia. --Lumbar puncture indicated for meningitis suspecion --Visual acuity testing used to monitor recovery.

A patient has recurrent epistaxis without localized signs of irritation. Which laboratory tests may be performed to evaluate this condition? Select all that apply. a. BUN and creatinine b. CBC with platelets c. Liver function tests d. PT and PTT e. PT/INR

B CBC with platelets D PT and PTT E PT/INR A CBC with platelets is part of the diagnostic workup along with coagulation studies. --LFTs and renal function tests aren't used to evaluate recurrent epistaxis.

Patients who meet the criteria for statin therapy to help prevent atherosclerotic cardiovascular disease are those with a history of a. a 10 year risk score of 5% and an LDL of 165 mg/dL. b. a 10 year risk score of 8% with an LDL of 80 mg/dL. c. a low-density lipoprotein (LDL) level >190 mg/dL. d. diabetes and an LDL between 40 and 70 mg/dL. e. previous myocardial infarction.

B a 10 year risk score of 8% with an LDL of 80 mg/dL. C a low-density lipoprotein (LDL) level >190 mg/dL E previous MI Patients with previous MI, those with risk scores >8% and an LDL >70 mg/dL, and those with LDL levels >190 mg/dL are candidates for statin therapy. -Patients with a risk score <7.5% with LDL levels between 75 and 190 mg/dL are not candidates and patients who have DM with LDL levels <75 mg/dL are not candidates.

Which are the causes of secondary HTN? a. Increased salt intake b. Isometric exercises c. NSAIDs d. Oral contraceptives e. Sleep apnea

C NSAIDs D Oral contraceptives E Sleep apnea NSAIDs and OCPs can both increase the risk of HTN. Sleep apnea causes secondary HTN. --Increased salt intake does not cause HTN, but those with HTN are more sensitive to sale. --Regular isometric exercise can decrease blood pressure.

1. During a preplacement screening for a person hired for a job requiring heavy lifting, a primary care provider notes that the new employee has environmental allergies, a history of gastroesophageal reflux disease (GERD), recurrent eczema, a previous history of an ankle fracture, and normal lower back strength and flexibility. A urine drug screen is negative. What will be included in the report to the employer? (Select all that apply.) a. GERD history b. History of allergies and eczema c. History of ankle fracture d. Lower back screening results e. Urine drug screening results

D Lower back screening results E Urine drug screening results Only findings related to the ability of the individual to perform position requirements for the job are included in the report. Other findings should not be included, even though they may need to be addressed.

During a pre-placement screening for a person hired for a job requiring heavy lifting, a PCP notes that the new employee has environmental allergies, a hx of GERD, recurrent eczema, a previous hx of an ankle fx, and normal lower back strength and flexibility. A urine drug screen is negative. What will be included in the report to the employer? Select all that apply. a. GERD history b. History of allergies and eczema c. History of ankle fx d. Lower back screening results e. Urine drug screening results

D Lower back screening results E Urine drug screening results Only findings related to the ability of the individual to perform position requirements for the job are included in the report. Other findings should not be included, even though they may need to be addressed.

An asymptomatic 63-year-old female has an LDL level of 135 mg/dL. Which test is beneficial to assess this patient's CAD risk? a. Coronary artery calcium score b. C-reactive protein c. Exercise echocardiography d. Myocardial perfusion imaging

EM CAD ANS: B C-reactive protein The CRP is useful in asymptomatic women >60 years who have LDL <160 mg/dL to predict CAD risk. --Although the CACS has shown some benefit in patients with moderate risk, the role for this diagnostic test is unclear. --Exercise echocardiography and myocardial perfusion imaging are not performed initially.

A patient develops acute bronchitis and is diagnosed as having influenza. Which medication will help reduce the duration of symptoms in this patient? a. Azithromycin b. Clindamycin c. Oseltamivir d. Trimethoprim-sulfamethoxazole

EM acute bronchitis ANS: C Oseltamivir (tamiflu) Oseltamivir is an antiviral medication used to reduce the severity and duration of symptoms of influenza. --The other medications are antibacterial medications and not effective to treat influenza.

Which risk assessment for CAD is recommended for all female patients? a. Coronary artery calcium score b. Electrocardiogram c. Exercise stress test d. Framingham risk score

EM CAD ANS: D Framingham risk score The Framingham risk score is a quick method for identifying potential risk for CAD and can guide providers in choosing subsequent tests based on risk level. --The ECG is performed on women with risk factors. --The exercise stress test is useful in symptomatic women who have a normal ECG. --The CACS may be used if moderate risk is present.

A patient with increased left-sided heart pressure will have which type of pulmonary hypertension? a. Group 2 b. Group 3 c. Group 4 d. Group 5

EM HTN ANS: A Group 2 Group 2 pulmonary hypertension is associated with increased left-sided heart pressure.

An African-American patient who is being treated with a thiazide diuretic for chronic HTN reports blurred vision and SOB. The provider notes B/P 185/115. What is the recommended action for this patient? a. Add a beta-blocker to the patient's regimen b. Admit to the hospital for evaluation and treatment c. Increase the dose of the thiazide medication d. Prescribe a calcium channel blocker

EM HTN ANS: B Admit to the hospital for evaluation and treatment Patients with B/P >180/120 or those with signs of target organ symptoms should be admitted to inpatient treatment with specialist consultation. --Changing the medications may be done with consultation, but a hospitalization and stabilization must be done initially.

A young adult patient is being treated for hypertension and is noted to have a resting blood pressure of 135/88 mm Hg just after finishing a meal. After standing, the patient has a blood pressure of 115/70 mm Hg. What is the likely cause of this change in blood pressure? a. A hyperglycemic episode b. Antihypertensive medications c. Neurogenic orthostatic hypotension d. Postpriandal hypotension

EM HTN ANS: B Antihypertensive medications Medications to treat HTN may cause orthostatic hypotension. --Hypoglycemia may cause hypotension. --Neurogenic orthostatic hypotension is less likely. --Postprandial (after meals) hypotension occurs in elderly patients.

A patient with pulmonary arterial hypertension (PAH) has increased dyspnea with activity. Which drug may be prescribed to manage this on an outpatient basis? a. An inhaled prostanoid b. Bosentan c. Epoprostenol d. Trepostinil

EM HTN ANS: B Bosentan Bosentan helps promote pulmonary artery smooth muscle cell proliferation and improves exercise capacity. It is also given PO, so is easy to give on an outpatient basis. Inhaled prostanoids have a short half-life and must be given 6 to 9 times daily. Epoprostenol has a short half-life and must be given IV. Trepostinil is given IV.REF: Management/Table 114—1: Therapies for Pulmonary Arterial Hypertension

A patient who has had mild pulmonary hypertension with a previous symptom of a loud second heart sound on exam now has edema and jugular vein distension. This indicates which complication? a. Left ventricular dysfunction b. Right ventricular dysfunction c. Tricuspid valve involvement d. Mitral valve involvement

EM HTN ANS: B Right ventricular dysfunction Right ventricular dysfunction occurs as the disease worsens with manifestations that include jugular vein distension, edema, and increased liver size. --These symptoms do not indicate left ventricular dysfunction or valvular involvement.

A 55-year-old patient has B/P 138/85 on three occasions. The patient denies headaches, palpitations, snoring, muscle weakness, and nocturia and does not take any medications. What will the provider do next to evaluate this patient? a. Assess serum cortisol levels b. Continue to monitor blood pressure at each health maintenance visit c. Order urinalysis, CBC, BUN, and creatinine d. Refer to a specialist for a sleep study

EM HTN ANS: C Order urinalysis, CBC, BUN, and creatinine This patient has pre-hypertension levels and should be evaluated. UA, CBC, BUN, and creatinine help to evaluate renal fxn and are in the initial workup. --Serum cortisol levels are performed if pheochromocytoma (a rare tumor that usually starts in the cells of one of your adrenal glands) is suspected, which would cause a HA. --The patient does not have snoring, so a sleep study is not indicated at this time. It is not correct to continue to monitor without assessing possible causes of early hypertension.

A patient who has undergone surgical immobilization for a femur fracture reports dyspnea and chest pain associated with inspiration. The patient has a heart rate of 120 beats per minute. Which diagnostic test will confirm the presence of a pulmonary embolism? a. Arterial blood gases b. CT angiography c. D-dimer d. Electrocardiogram

EM PE ANS: B CT angiography Ct angiography is used to diagnose PE. --D-dimer assays have good negative predictive value but have poor positive predictive value, making it useful for excluding but not confirming the presence of PE. --An ECG does not confirm PE but is used to demonstrate comorbid conditions. --ABGs do not confirm PE & are used to identify the degree of respiratory compromise.

A patient develops a pulmonary embolism after surgery and shows signs of right-sided heart failure. Which drug will be administered to this patient? a. Low molecular heparin b. Tissue plasminogen activator c. Unfractionated heparin d. Warfarin

EM PE ANS: B Tissue plasminogen activator Fibrinolytic therapy with rtPA (process that prevents blood clots from growing/breaks down the clot) is given to those with hypotension & R heart failure. --Heparin is used for its anticoagulant properties in all patients with PE. --Warfarin is not indicated.

Which clinical sign is especially worrisome in a patient with a pulmonary embolism? a. Abnormal lung sounds b. Dyspnea c. Hypotension d. Tachycardia

EM PE ANS: C Hypotension Hypotension in a patient with PE has a high correlation with acute right ventricular failure and subsequent death. --The other signs are common with PE.

A patient is recovering from Mycoplasma pneumoniae infection and has a persistent cough 6 weeks after the infection. What will the provider do? a. Perform CXR to assess for secondary infection b. Perform PFT and asthma challenge testing c. Prescribe a second round of azithromycin to treat the persistent infection d. Reassure the patient that this is common after M. pneumoniae infection

EM Pneumonia ANS: D Reassure the patient that this is common after M. pneumoniae infection Post-infection cough is common after M. pneumoniae infection and may persist up to 8 weeks after the infection; this type of cough generally needs no intervention. --It is not necessary to perform CXR unless secondary infection is suspected. --Abx are not indicated. --Unless the cough persists after 8 weeks, asthma testing is not indicated.REF:

A patient who has asthma calls the provider to report having a peak flow measure of 75%, shortness of breath, wheezing, and cough, and tells the provider that the symptoms have not improved significantly after a dose of albuterol. The patient uses an inhaled corticosteroid medication BID. What will the provider recommend? a. Administering two more doses of albuterol b. Coming to the clinic for evaluation c. Going to the emergency department d. Taking an oral corticosteroid

EM asthma ANS: A Administering two more doses of albuterol The patient is experiencing an asthma exacerbation and should follow the asthma action plan (AAP) which recommends three doses of albuterol before reassessing. --The peak flow is above 70%, so ED admission is not indicated. 2nd) The patient may be instructed to come to the clinic for oxygen saturation and spirometry evaluation after administering the albuterol. 3rd) An oral corticosteroid may be prescribed if the patient will be treated as an outpatient after following the AAP.

A patient is seen in clinic for an asthma exacerbation. The provider administers three nebulizer treatments with little improvement, noting a pulse oximetry reading of 90% with 2 L of oxygen. A peak flow assessment is 70%. What is the next step in treating this patient? a. Administer three more nebulizer treatments and reassess b. Admit to the hospital with specialist consultation c. Give epinephrine injections and monitor response d. Prescribe an oral corticosteroid medication

EM asthma ANS: B Admit to the hospital with specialist consultation Patients having an asthma exacerbation should be referred if they fail to improve after 3 nebulizer treatments or three epinephrine injections, have a peak flow less than 70% and a pulse oximetry reading less than 90% on room air. --Giving more nebulizer treatments or administering epinephrine are not indicated. --The patient will most likely be given IV corticosteroids; oral corticosteroids would be given if the patient is managed as an outpatient.

A young adult patient develops a cough persisting longer than 2 months. The provider orders PFTs and a CXR, which are normal. The patient denies abdominal complaints. There are no signs of rhinitis or sinusitis and the patient does not take any medications. What will the provider evaluate next to help determine the cause of this cough? a. 24-hour esophageal pH monitoring b. Methacholine challenge test c. Sputum culture d. Tuberculosis testing

EM asthma ANS: B Methacholine challenge test Chronic cough without other symptoms may indicate asthma. If PFTs are normal, a methacholine challenge test may be performed. --24-hour esophageal pH monitoring is sometimes performed to evaluate for GERD, but this patient does not have abdominal symptoms and this test is usually not performed because it is inconvenient. --Sputum culture is not indicated. TB is less likely.

2. Which symptom in a patient diagnosed with asthma indicates severe bronchospasm? a. Breathlessness with minimal activity or eating b. Pausing to breathe while attempting to talk c. Repetitive, spasmodic coughing at night d. Wheezing after exposure to a trigger

EM asthma ANS: B Pausing to breathe while attempting to talk Inability to speak a full sentence without pausing to breathe indicates severe bronchospasm. Breathlessness, repetitive and spasmodic coughing, and wheezing are all common signs of bronchospasm and do not necessarily indicate severe bronchospasm

A patient reports SOB with activity and exhibits increased work of breathing with prolonged expirations. Which diagnostic test will the provider order to confirm a diagnosis in this patient? a. Arterial blood gases b. Blood cultures c. Spirometry d. Ventilation/perfusion scan

EM asthma ANS: C Spirometry The patient has signs of either asthma or COPD. Spirometry is essential to both the diagnosis and management of both. --ABGs are useful when evaluating the severity of exacerbations, but are not specific to these diseases. --Blood cultures are drawn if pneumonia is suspected. --A ventilation/perfusion scan is performed to evaluate for pulmonary thromboembolic disease.

An adult develops chronic cough with episodes of wheezing and SOB. The provider performs CXR and other tests to r/o infection, upper respiratory, and gastroesophageal causes. Which test will the provider order initially to evaluate the possibility of asthma as the cause of these symptoms? a. Allergy testing b. Methacholine challenge test c. Peak expiratory flow rate d. Spirometry

EM asthma ANS: D Spirometry Spirometry is recommended at the time of initial assessment to confirm the diagnosis of asthma. --Allergy testing is performed only if allergies are a possible trigger. --The methacholine challenge test is performed if spirometry is inconclusive. --PEFR is generally used to monitor asthma symptoms.

1. A patient diagnosed with asthma has been prescribed three bronchodilator treatments but continues to experience wheezing and SOB. The health care provider caring for the patient notes an oxygen saturation of 90% on room air. What action is indicated? a. Administer oxygen and continue to monitor the patient. b. Contact the respiratory therapist to administer another treatment. c. Notify the patient's physician immediately. d. Reassure the patient that the treatments will take effect soon.

EM asthsma ANS: C Notify the patient's physician immediately. Patients with bronchospasm who have oxygen saturations less than 92% on room air and who fail to improve with nebulizer treatment given three times, need physician consultation. While oxygen administration and further nebulizer treatments may be indicated, it is incorrect to continue to monitor the patient without notifying the physician.

3. A patient reports using artificial tears for comfort because of burning and itching in both eyes, but reports worsening symptoms. The provider notes redness and discharges along the eyelid margins with clear conjunctivae. What is the recommended treatment? a. Abx drops QID b. Compresses, lid scrubs, and abx ointment c. Oral abx given prophylactically for several months d. Reassurance that this is a self-limiting condition

EM blephartis ANS: B Compresses, lid scrubs, and abx ointment This patient has symptoms of blepharitis without conjunctivitis. Initial treatment involves lid hygiene and antibiotic ointment may be applied after lid scrubs. --An abx solution is used if conjunctivitis is present. --Oral abx are used in severe cases. *This disorder is generally chronic.

2. When screening for metastatic cancer in a patient with lung cancer, the provider will assess for Select all that apply. a. complaints of headache. b. increased cough. c. low hematocrit. d. lymph nodes greater than 1 cm. e. unexplained weight gain.

EM cancer A complaints of headache C low hematocrit D lymph nodes greater than 1 cm HA may indicate brain metastases. Low hematocrit and lymphadenopathy with nodes greater than 1 cm also indicate metastasis. --Increased cough is a sign of lung cancer itself, not metastasis. --Patients with metastatic cancer have unexplained weight loss of more than 10 pounds.

During a routine physical examination, a provider notes a shiny, irregular, painless lesion on the top of one ear auricle and suspects skin cancer. What will the provider tell the patient about this lesion? a. A biopsy should be performed. b. Immediate surgery is recommended. c. It is benign and will not need intervention. d. This is most likely malignant.

EM cancer ANS: A A biopsy should be performed This lesion is characteristic of basal cell carcinoma, which is slow-growing cancer least likely to metastasize. A biopsy should be performed to evaluate this. Until a biopsy is performed, the provider cannot determine whether it is benign. --Immediate surgery is not necessary.

A patient with a cough has a suspicious lung lesion, a mediastinal lymph mass, and several bone lesions. What test is indicated to determine the histology and staging of this cancer? a. Biopsy of a bone lesion b. Bone marrow aspiration and biopsy c. Bronchoscopy with lung biopsy d. Thoracentesis and pleural fluid cytology

EM cancer ANS: A Biopsy of a bone lesion The diagnosis and stage should be determined in the least invasive manner possible. A single biopsy of the bone lesion can determine histology and staging. --The other procedures are more invasive and not necessary.

A 50-year-old female patient has a blood pressure of 118/72 mm Hg, a negative family history for breast and ovarian cancer, a normal Pap smear 2 years prior, and a Framingham risk screening within normal limits. Which should be part of this patient's routine annual well-patient exams? a. Bone mineral density screening b. Breast cancer screening and mammogram c. Cervical cancer screening with a Pap test d. Lipid screening and cholesterol tests

EM cancer ANS: B Breast cancer screening and mammogram The ACS recommends yearly mammograms at 40. DEXA begins at 50 to 64, based on risk. --Cervical cancer screening every 3 years unless there is increased risk. --Lipid screening and cholesterol annually if there is increased risk according to the Framingham guidelines.

A patient with small cell lung cancer (SCLC) has undergone chemotherapy with a good initial response to therapy. What will the provider tell this patient about the prognosis for treating this disease? a. Surgical resection will improve survival chances. b. That relapse is likely with a poor prognosis. c. There is an 80% chance of 5-year survival. d. Treatment will proceed with curative intent.

EM cancer ANS: B That relapse is likely with a poor prognosis Although SCLC often responds very well initially to chemotherapy, the majority of patients will relapse and the 5-year survival rates are approximately 10%. --Surgical resection does not play a significant role in the management of SCLC because the majority of patients have metastatic disease at diagnosis. --Treatment is generally palliative.

2. A patient reports bilateral reports burning and itching eyes for several days. The provider notes a boggy appearance to the conjunctivae, along with clear, watery discharge. The patient's eyelids are thickened and discolored. There are no other symptoms. Which type of conjunctivitis is most likely? a. Allergic b. Bacterial c. Chemical d. Viral

EM conjunctivitis ANS: A Allergic generally presents simultaneously in both eyes with itching as a predominant feature. Discharge is generally clear or stringy and white and the patient will have lid discoloration, thickening, and erythema. --Bacterial conjunctivitis is characterized by acute inflammation of the conjunctivae along with purulent discharge. --Chemical conjunctivitis will not have purulent discharge. --Viral conjunctivitis is usually in association with a URI.

1. A patient who has a cold develops conjunctivitis. The provider notes erythema of one eye with profuse, watery discharge, and enlarged anterior cervical lymph nodes, along with a fever. Which treatment is indicated? a. Antihistamine-vasoconstrictor drops b. Artificial tears and cool compresses c. Topical antibiotic eye drops

EM conjunctivitis ANS: B Artificial tears and cool compresses Viral conjunctivitis accompanies URI and is generally self-limited, lasting 5 to 14 days. Symptomatic treatment is recommended. --Antihistamine-vasoconstrictor drops are used for allergic conjunctivitis. --Topical antibiotic drops are sometimes used for bacterial conjunctivitis. --Topical corticosteroid drops are used for severe inflammation.

3. A patient with allergic conjunctivitis who has been using topical antihistamine-vasoconstrictor medication reports worsening symptoms. What is the provider's next step in managing this patient's symptoms? a. Consider prescribing a topical mast cell stabilizer b. Determine the duration of treatment with this medication c. Prescribe a non-sedating oral antihistamine d. Refer the patient to an ophthalmologist for further care

EM conjunctivitis ANS: B Determine the duration of treatment with this medication 1st) Antibiotic-vasoconstrictor agents can have a rebound effect with worsening symptoms if used longer than 3 to 7 days, so the provider should determine whether this is the cause. 2nd) Topical mast cell stabilizers are useful as prophylaxis for recurrent or persistent allergic conjunctivitis. 3rd) Oral antihistamines may be the next step if it is determined that the cause of worsening symptoms is related to the allergy. It is not necessary to refer to ophthalmology at this time.

A child has unilateral eyelid edema, warmth, & erythema and does not exhibit pain with ocular movement. Which is most likely true about this child's infection? a. Decreased visual acuity may occur b. Increased intraocular pressure will be present c. Optic nerve compromise is a complication d. The eye is typically spared without conjunctivitis

EM conjunctivitis ANS: D The eye is typically spared without conjunctivitis This child has symptoms of preseptal cellulitis (infection of the eyelids and surrounding soft tissues that are anterior to the orbital septum) in which the eye is typically spared. --The other findings are consistent with orbital cellulitis.

Which patients should be referred immediately to an ophthalmologist after an eye injury and initial treatment? Select all that apply. a. A patient who was sprayed by lawn chemicals b. A patient who works in a metal fabrication shop c. A patient with a corneal abrasion d. A patient with a full-thickness corneal laceration e. A patient with irritation secondary to wood dust

EM corneal surface defects A A patient who was sprayed by lawn chemicals B A patient who works in a metal fabrication shop D A patient with a full-thickness corneal laceration Patients with chemical eye injuries, any with possible metallic foreign bodies, and those with full-thickness corneal lacerations must have an immediate referral. --Corneal abrasions and irritation from wood dust may be managed by primary care providers.

A primary care provider may suspect cataract formation in a patient with which finding? a. Asymmetric red reflex b. Corneal opacification c. Excessive tearing d. Injection of conjunctiva

EM corneal surface defects ANS: A Asymmetric red reflex --Corneal opacification, excessive tearing, and corneal injection are not symptoms of cataracts.

A patient who works in a furniture manufacturing shop reports a sudden onset of severe eye pain while sanding a piece of wood and now has copious tearing, redness, and light sensitivity in the affected eye. On examination, the conjunctiva appears injected (an excess of blood in the vessels), but no foreign body is visualized. What is the practitioner's next step? a. Administration of antibiotic eye drops b. Application of topical fluorescein dye c. Instillation of cycloplegic eye drops d. Irrigation of the eye with normal saline

EM corneal surface defects ANS: B Application of topical fluorescein dye The practitioner must determine if there is a corneal abrasion and will instill fluorescein dye in order to examine the cornea under a Wood's lamp (a small handheld device that uses a black light to illuminate areas of your skin). --Antibiotic eye drops are not indicated as initial treatment. --Cyclopedic drops are used occasionally for pain control but should be used with caution. --Irrigation of the eye is indicated for chemical burns.

A patient has an elevated, yellowish-white lesion adjacent to the cornea at the 3 o'clock position of the right eye. The provider notes pinkish inflammation with dilated blood vessels surrounding the lesion. What will the provider tell the patient about this lesion? a. Artificial tear drops are contraindicated b. Spontaneous bleeding is likely c. UVB eye protection is especially important d. Visine may be used for symptomatic relief

EM corneal surface defects ANS: C UVB eye protection is especially important This patient has a pinguecula which has become inflamed. Wide-brimmed hats and sunglasses with UVB protection should be advised since UVB light will make this worse. --Artificial tear drops are recommended to reduce irritation. --These types of lesions typically do not bleed spontaneously. Visine is contraindicated because chronic vasoconstriction may lead to rebound inflammation.

Which are risk factors for developing hearing loss caused by presbycusis? Select all that apply. a. DM b. GERD c. HTN d. Liver disease e. Smoking

EM hearing loss A DM C HTN E Smoking Presbycus is a gradual degeneration within the cochlea that accompanies aging. DM, HTN, and smoking may hasten these changes. --GERD & liver disease are not associated with an increased rate of changes.

A child who has recurrent otitis media fails a hearing screen at school. The provider suspects which type of hearing loss in this child? a. Central b. Conductive c. Mixed type d. Sensorineural

EM hearing loss & otitis ANS: B Conductive A common cause of conductive loss is fluid in the middle ear as a result of chronic otitis media with effusion. --Central hearing loss is related to CNS disorders. --Mixed type hearing loss is r/t the causes of both conductive and sensorineural hearing loss. --Sensorineural hearing loss is caused by damage to the structures in the inner ear, usually caused by infection, barotrauma, or trauma.

A patient has infective endocarditis and is being treated with empiric antibiotics after blood cultures are inconclusive. The patient develops a severe headache along with transient neurologic changes. What is the likely cause of these symptoms? a. Extra-cardiac abscess formation b. Haemophilus infection c. Mycotic aneurysm d. Rheumatic heart fever

EM infective endocarditis ANS: C Mycotic aneurysm Patients with mycotic aneurysms will present with symptoms of severe unrelenting headache, neurological changes, and signs of cranial nerve involvement. --Extracardiac abscess formation depends on the organ involved. --Haemophilus infections cause larger vegetations in the heart. --Rheumatic heart fever has a classic group of symptoms involving the skin.

A patient who is on renal dialysis is diagnosed with infective endocarditis. What causative organisms are more likely in this patient? a. Enterococcal organisms b. Neisseria gonorrhea c. Pseudomonas aeruginosa d. Staphylococcus aureus

EM infective endocarditits ANS: D Staphylococcus aureus This patient is more likely to have a health care associated endocarditis; most of these are caused by S. aureus. --Enterococcal organisms are the second highest cause in this population.

A 55-year-old patient who had influenza in the previous influenza season asks about the flu vaccine. What will the provider tell the patient? a. Having influenza vaccine confers lifetime immunity b. The FluMist vaccine may be used each year c. The Fluzone High-Dose vaccine is recommended d. The trivalent influenza vaccine is indicated annually

EM influenza vaccine ANS: D The trivalent influenza vaccine is indicated annually Because the strains of influenza vary from year to year, annual immunization with TIV is indicated for all persons. --Lifetime immunity is not conferred with infection. --The Flu Mist is used in persons 50 years of age and younger. --The Fluzone High-Dose vaccine is used in patients older than 65 year.

Which symptoms in children are evaluated using a parent-reported scoring system to determine the severity of pain in children with otitis media? Select all that apply. a. Appetite b. Difficulty sleeping c. Level of cooperation d. Poor hearing e. Tugging on ears

EM otitis A Appetite B Difficulty sleeping E Tugging on ears Decreased appetite, difficulty sleeping, and tugging on ears are part of the Acute Otitis Media Severity of Symptom Scale used to evaluate pediatric pain. --Children may refuse to cooperate for reasons other than pain. --Poor hearing is not part of the pain assessment.

A patient reports a feeling of fullness and pain in both ears and the practitioner elicits exquisite pain when manipulating the external ear structures. What is the likely diagnosis? a. Acute otitis externa b. Acute otitis media c. Chronic otitis externa d. Otitis media with effusion

EM otitis ANS: A Acute otitis externa This patient's symptoms are classic for acute otitis externa. --Chronic otitis externa more commonly presents with itching. --Acute otitis media is accompanied by fever and tympanic membrane inflammation, but not external canal inflammation. --Otitis media with effusion causes a sense of fullness but not pain.

A patient has an initial episode otitis external associated with swimming. The patient's ear canal is mildly inflamed and the tympanic membrane is not involved. Which medication will be ordered? a. Cipro HC b. Fluconazole c. Neomycin d. Vinegar and alcohol

EM otitis ANS: A Cipro HC In the absence of a culture, the provider should choose a medication that is effective against both P. aeruginosa and S. aureus. Cipro HC covers both organisms and also contains a corticosteroid for inflammation. --Fluconazole is an oral anti-fungal medication used when fungal infection is present. --Neomycin alone does not cover these organisms. --Vinegar and alcohol is used to treat mild fungal infections.

Which are risk factors for developing otitis externa? Select all that apply. a. Cooler, low-humidity environments b. Exposure to someone with otitis externa c. Having underlying DM d. Use of earplugs and hearing aids e. Vigorous external canal hygiene

EM otitis C Having underlying DM D Use of earplugs and hearing aids E Vigorous external canal hygiene Otitis externa is a cellulitis of the external canal that develops when the integrity of the skin is compromised. --DM predisposes patients to skin disorders. --Using devices that cause moisture retention and irritation will increase the risk. --Vigorous cleansing removes protective cerumen. --Warm, high-humidity environments increase risk. --The disease is not contagious.

A patient is diagnoses with peritonsillar abscess and will be hospitalized for intravenous antibiotics. What additional treatment will be required? a. Intubation to protect the airway b. Needle aspiration of the abscess c. Systemic corticosteroid administration d. Tonsillectomy and adenoidectomy

EM peritonsillar abcess ANS: B Needle aspiration of the abscess Needle aspiration, antibiotics, pain medication, and hydration can effectively treat peritonsillar abscess. --Intubation is not performed unless the airway is compromised. --Systemic corticosteroid administration is useful, but not required in all cases. --Tonsillectomy alone is sometimes performed if recurrent tonsillitis or peritonsillar abscess is present.

An adolescent has fever, chills, and a severe sore throat. On exam, the provider notes foul-smelling breath and a muffled voice with marked edema and erythema of the peritonsillar tissue. What will the primary care provider do? a. Evaluate for possible epiglottitis b. Perform a rapid strep and throat culture c. Prescribe empiric oral antibiotics d. Refer the patient to an otolaryngologist

EM peritonsillar abscess ANS: D Refer the patient to an otolaryngologist This patient has clinical signs of peritonsillar abscess, which may be diagnosed on clinical signs alone. Patients with peritonsillar abscess should be referred to an otolaryngologist for possible I&D of the abscess and hospitalization for IV antibiotics. A rapid strep and culture are not indicated. Oral antibiotics generally do not work. REF: Clinical Presentation/Management

A patient has a cough and fever and the provider auscultates rales in both lungs that do not clear with cough. The patient reports having a headache and sore throat prior to the onset of coughing. A CXR shows patchy, nonhomogeneous infiltrates. Based on these findings, which organism is the most likely cause of this patient's pneumonia? a. A virus b. Mycoplasma c. S. pneumoniae d. Tuberculosis

EM pneumonia ANS: B Mycoplasma Atypical pneumonia (mycoplasma) often present with headache and sore throat and will have larger areas of infiltrate on chest radiograph. --Viral pneumonia show more diffuse radiographic findings. --S. pneumonia will have high fever and cough and distinct areas of infiltration.

A young adult, previously healthy clinic patient has symptoms of pneumonia including high fever and cough. Auscultation reveals rales in the left lower lobe. A CXR is normal. The patient is unable to expectorate sputum. Which treatment is recommended for this patient? a. A B-lactam antibiotic plus a fluoroquinolone b. A respiratory fluoroquinolone antibiotic c. Empiric treatment with a macrolide antibiotic d. Hospitalization for intravenous antibiotics

EM pneumonia ANS: C Empiric (educated guess) treatment with a macrolide antibiotic This patient likely has CAP. The patient has typical symptoms and, even though the CXR is normal, will require outpatient treatment. For CAP in a previously healthy individual, treatment with a macrolide antibiotic is the recommended first-line therapy. --B-lactam plus fluoroquinolone therapy is used for patients in the ICU. --Respiratory fluoroquinolones are used for patients with underlying disorders who develop pneumonia. --Hospitalization is not necessary.

A patient has recurrent sneezing, alterations in taste and smell, watery, itchy eyes, and thin, clear nasal secretions. The provider notes puffiness around the eyes. The patient's vital signs are normal. What is the most likely diagnosis for this patient? a. Acute sinusitis b. Allergic rhinitis c. Chronic sinusitis d. Viral rhinitis

EM rhinitis ANS: B Allergic rhinitis This is typically allergic rhinitis. --Sinusitis causes facial pain, fever, and purulent discharge. --Viral rhinitis will also cause purulent discharge and other symptoms of URI.

A patient is concerned about frequent nasal stuffiness and congestion that begins shortly after getting out of bed in the morning. The patient denies itching and sneezing. A physical examination reveals erythematous nasal mucosa with scant watery discharge. What treatment will the provider recommend for this patient? a. Consultation for immunotherapy b. Daily intranasal steroids c. Oral antihistamines each morning d. Oral decongestants as needed

EM rhinitis ANS: B Daily intranasal steroids This patient has symptoms of vasomotor or idiopathic rhinitis. Intranasal steroids are an effective treatment. --Immunotherapy is not effective. --This type of rhinitis typically does not respond to antihistamines. --Oral decongestants are effective but are best used around the clock, not just prn.

A patient has seasonal rhinitis symptoms and allergy testing reveals sensitivity to various trees and grasses. What is the first-line treatment for this patient? a. Antihistamine spray b. Intranasal cromolyn c. Intranasal steroids d. Oral antihistamines

EM rhinitis ANS: C Intranasal steroids Intranasal steroids are the mainstay of treatment and are the most effective medication for preventing symptoms. --Antihistamine sprays are helpful, but are not first-line treatments. --Intranasal cromolyn can be effective, but must be used four times daily. --Oral antihistamines are used in conjunction with intranasal steroids, but are less effective than the steroids.

A pregnant woman develops nasal congestion with chronic nasal discharge. What is the recommended treatment for this patient? a. Intranasal corticosteroids b. Prophylactic antibiotics c. Saline lavage d. Topical decongestants

EM rhinitis ANS: C Saline lavage Saline lavage is recommended for pregnancy rhinitis; the condition will resolve after delivery. --There is no human data on the safety of intranasal corticosteroids during pregnancy. --Prophylactic antibiotics are not indicated; this is not an infectious condition. --Topical decongestants can cause rebound symptoms.

A provider determines that a patient has chronic rhinosinusitis without nasal polyps. What is the first-line treatment for this condition? a. Intranasal corticosteroids b. Oral decongestants c. Systemic corticosteroids d. Topical decongestants

EM rhinitis and sinusitis ANS: A Intranasal corticosteroids Intranasal corticosteroids are the mainstay of treatment for CRS. --Oral decongestants should be used sparingly, only when symptoms are intolerable. --Topical decongestants can cause rebound symptoms. --Systemic steroids are not indicated.

A patient reports persistent nasal blockage, nasal discharge, and facial pain lasting on the right side for the past 4 months. There is no history of sneezing or eye involvement. The patient has a history of seasonal allergies and takes a non-sedating antihistamine. What does the provider suspect is the cause of these symptoms? a. Allergic rhinitis b. Autoimmune vasculitides c. Chronic rhinosinusitis d. Rhinitis medicamentosa

EM rhinitis and sinusitis ANS: C Chronic rhinosinusitis present when symptoms occur longer than 3 months. --Sneezing and itchy, watery eyes tend to occur with allergic rhinitis. --Autoimmune vasculitides affects upper and lower respiratory tracts as well as the kidneys. --Rhinitis medicamentosa occurs with use of nasal decongestants and not oral antihistamines.

A patient has nasal congestion, fever, purulent nasal discharge, headache, and facial pain and begins treatment with amoxicillin-clavulanate. At a follow-up visit 10 days after initiation of treatment, the patient continues to have purulent discharge, congestion, and facial pain without fever. What is the next course of action for this patient? a. A CT scan of the paranasal sinuses b. A referral to an otolaryngologist c. A second course of amoxicillin-clavulanate d. A trial of azithromycin

EM sinusitis ANS: C A second course of amoxicillin-clavulanate This patient may have subacute sinusitis and may benefit from a second course of the antibiotic. --The lack of fever shows improvement, so this antibiotic may be used. --CT scan is usually not performed in adults unless other complications are present or suspected. --Referral to an otolaryngologist is necessary if no improvement after the second course of antibiotics. --Azithromycin is not used in adults unless pregnant, due to resistance patterns.

An adult patient with a history of recurrent sinusitis and allergic rhinitis reports chronic tearing in one eye, ocular discharge, and eyelid crusting. The provider suspects nasolacrimal duct obstruction. Which initial treatment will the provider recommend? a. Antibiotic eye drops b. Nasolacrimal duct probing c. Systemic antibiotics d. Warm compresses

EM sinusitis ANS: D Warm compresses This is most likely acquired nasolacrimal duct obstruction. 1st) Initial treatment should include warm compresses. 2nd) Abx only if an infection is present. 3rd) definitive treatment usually requires surgery. --Nasolacrimal duct probing (opens valves btwn nasolacrimal duct & nose) not useful for acquired conditions

Which are potential complications of chronic or recurrent sinusitis? Select all that apply. a. Allergic rhinitis b. Asthma c. Meningitis d. Orbital infection e. Osteomyelitis

EM sinusitis C Meningitis D Orbital infection E Osteomyelitis Complications of chronic or recurrent sinusitis include the spread of infection to other tissues and may cause meningitis, orbital cellulitis, and osteomyelitis. --Allergic rhinitis and asthma are associated with chronic sinusitis, but not complications of this condition.

During a preparticipation sports physical, the examiner notes a difference in strength of the patient's radial and femoral pulses with the femoral pulses being weaker. What will the provider do? a. Evaluate for orthostatic hypotension. b. Obtain Doppler studies of lower extremity circulation. c. Reassure the patient that this is a normal finding. d. Refer the patient for a cardiologic exam.

EM sports physical ANS: D Refer the patient for a cardiologic exam Differences in strength between radial and femoral pulses may indicate coarctation of the aorta and should be evaluated by a cardiologist. --This finding does not indicate orthostatic hypotension. The likelihood of decreased circulation is low in a young athlete.

Which symptoms may occur with vestibular neuritis? Select all that apply. a. Disequilibrium b. Fever c. Hearing loss d. Nausea and vomiting e. Tinnitus

EM tinitis A Disequilibrium D Nausea and vomiting E Tinnitus Vestibular neuritis can cause severe vertigo, disequilibrium, nausea, vomiting, and tinnitus --not fever or hearing loss.

1. A patient who has a history of working around asbestos and silica fibers is concerned about developing lung disease. The primary care provider determines that the patient has a previous history of asthma as a child and currently has frequent episodes of bronchitis. A PE is normal and PFTs and radiographs are negative. What action is correct? a. Reassure the patient about the normal findings b. Refer the patient to an occupational health specialist c. Request a workplace environmental assessment d. Suggest that the patient follow up with a pulmonologist

Refer the patient to an occupational health specialist Patients with environmental exposure may not have symptoms or positive findings. Because this patient reports frequent bronchitis, this should be followed up with an occupational health specialist who can evaluate the degree of exposure and perform further testing. Normal findings are not necessarily reassuring. The occupational specialist may request an environmental assessment. Pulmonologists are not trained in occupational health. REF: Occupational Respiratory Diseases

A patient reports a sudden onset of sore throat, fever, malaise, and cough. The provider notes mild erythema of the pharynx and clear rhinorrhea without cervical lymphadenopathy. What is the most likely cause of these symptoms? a. Allergic pharyngitis b. Group A streptococcus c. Infectious mononucleosis d. Viral pharyngitis

exam matrix pharyngitis ANS: D Viral pharyngitis Viral pharyngitis will cause sore throat, fever, and malaise and is often accompanied by URI symptoms of cough and runny nose. --Allergic pharyngitis usually also causes dryness. --GAS causes high fever, cervical adenopathy, and marked erythema with exudate. --Infectious mononucleosis will cause an exudate along with cervical adenopathy.


Conjuntos de estudio relacionados

ACCT 470 Exam 2 Multiple Choices

View Set

Health Assessment Heart + Vessels

View Set

Micro--Chapter 20 Study Questions

View Set

Texas Real Estate Property Management

View Set